You are on page 1of 68

ANATOMY (Questions)

EMBRYOLOGY 9. True diverticulum is: March 2010


1. Umbilical cord contains: September 2005 a. Zenker’s diverticulum
b. Meckel’s diverticulum
a. 2 arteries and 2 veins
c. Duodenal diverticulum
b. 1 artery and 2 veins
d. Bladder diverticulum
c. 1 artery and 1 vein
10. Which of the following artery is present in
d. 2 arteries and 1 vein
anatomical snuff box: September 2010
2. Ligamentum arteriosum is derived from: March 2005
a. Anterior interosseous artery
a. Ductus arteriosus
b. Brachial artery
b. Ductus venosus
c. Radial artery
c. Ductus utriculosaccularis
d. Ulnar artery
d. Ductus reunions
11. Which of the following is not a constituent of
3. Arch of aorta develops from: September 2006 umbilical cord: March 2011
a. Right fourth aortic arch artery a. Wharton’s jelly
b. Left third aortic arch artery b. 2 arteries and 1 vein
c. Left fourth aortic arch artery c. Cloacal duct
d. Right third aortic arch artery d. Allantois
4. Lens is derived from: March 2007, September2010 12. Function of umbilical vein is: March 2011
a. Endoderm a. Carry oxygenated blood away from the fetus
b. Surface ectoderm b. Carry oxygenated blood towards the fetus
c. Mesoderm c. Carry deoxygenated blood away from the fetus
d. None of the above d. Carry deoxygenated blood towards the fetus
5. Fate of notochord is: September 2008 13. Ligamentum teres is: March 2011
a. Annulus fibrosis a. Remnant of ductus arteriosus
b. Nucleus pulposus b. Remnant of umbilical vein
c. Vertebral foramen c. Remnant if ductus venosus
d. Spinous process d. Remnant of umbilical artery
6. Trigone of urinary bladder develops from: March 2009 14. Anatomical obliteration of ductus arteriosus occurs
a. Mesoderm at: September 2011
b. Ectoderm a. Birth
c. Endoderm of urachus b. 3-4 day
d. None of the above c. 10 day
d. 30 day
7. In the adult heart, floor of fossa ovalis represents:
15. Which is not associated with vitello-intestinal duct:
March 2009
March 2011
a. Septum intermedium
a. Ileal diverticulum
b. Septum primum
b. Umbilical fistula
c. Septum spurium
c. Enterocystoma
d. Septum secundum
d. Mesenteric cyst
8. Which part of the ear has origin from all the three
16. Implantation (of blastocyst) occurs on which day
layers of germ layer: March 2010
after fertilization: September 2011
a. Auricle
a. 12
b. Tympanic membrane b. 10
c. Ossicles c. 8
d. Middle ear cavity d. 6
Self Assessment & Review of FMGE / MCI Screening Examination
2

17. Sperm attains motility in: September 2011 25. Blood testes barrier is formed by the: September 2009
a. Vas deferens a. Leydig cells
b. Rete testis b. Sertoli cells
c. Seminal vesicle c. Germ cells
d. Epididymis d. All of the above
18. Clitoris develops from which of the following: 26. Fenestrated capillaries are found in all except :
September 2011 September 2009
a. Genital tubercle a. Renal glomeruli
b. Genital ridge b. Intestinal villi
c. Wolffian duct c. Pancreas
d. Mullerian duct d. Muscle

HISTOLOGY 27. Goblet cells are not seen in: September 2009
a. Colon
19. Mucous membrane of vagina is lined by:
b. Trachea
September 2005
c. Conjunctiva
a. Nonkeratinized Stratified Squamous epithelium
d. Esophagus
b. Keratinized Stratified Squamous epithelium
c. Columnar cells 28. Peyer’s patches are present in: September 2010
d. Cuboidal epithelium a. Duodenum
b. Jejunum
20. Gland of Brunner’s is found in: September 2005
ANATOMY

c. Ileum
a. Jejunum
d. Stomach
b. Duodenum
c. Ileum 29. Periarteriolar lymphoid sheaths are seen in which
d. All organ: September 2011
21. Not a layer of retina: September 2007 a. Liver
a. Outer limiting membrane b. Spleen
b. Retinal pigment epithelium c. Kidney
c. Bowman’s membrane d. Heart
d. Ganglion cell layer 30. Uterus, before menarche, is lined by: September 2011
22. True about cardiac muscles are all except: a. Ciliated columnar epithelium
September 2007, September 2010 b. Stratifies squamous non-keratinized epithelium
a. Property of spontaneous and rhythmic contraction c. Startifies squamous keratinized epithelium
b. Cardiac muscle exhibits cross striations d. Cuboidal epithelium
c. Cardiac muscle cells are linear and longitudinal SUPERIOR EXTREMITY
d. Cardiac muscle is supplied by autonomic nerve 31. Root value of the Radial nerve is: September 2005
fibers a. C5, C6
23. Simple cuboidal epithelium is seen in: September 2008 b. C5, C6, C7
a. Skin c. C5, C6, C7, C8
b. Trachea d. C5, C6, C7, C8, T1
c. Oesophagus 32. Which of the following is an artery of the forearm:
d. Thyroid gland September 2006
24. Hassal’s corpuscles are seen in: September 2009 a. Brachial artery
a. Thymus b. Axillary artery
b. Spleen c. Ulnar artery
c. Bone marrow d. Femoral artery
d. Lymph node
Anatomy
3

33. Insertion of pectoralis major is at: September 2006 41. The ligament which transfers weight of arm to the
a. Lateral lip of bicipital groove of humerus trunk: March 2009
b. Medial lip of bicipital groove of humerus a. Costoclavicular ligament
c. In the bicipital groove of humerus b. Coracoclavicular ligament
c. Coracoacromial ligament
d. Clavicle
d. Coracohumeral ligament
34. Labourers nerve is another name for which nerve:
42. Nerve lying in the spiral groove of humerus is:
March 2007 a. Musculocutaneous nerve September 2009
a. Median nerve b. Ulnar nerve
b. Radial nerve c. Radial nerve
c. Ulnar nerve d. Median nerve
d. Axillary nerve 43. Bone, which usually does not have a medullary
35. Nerve roots involved in Erb’s palsy: September 2009 cavity: March 2011
a. C5, C6 a. Fibula
b. Clavicle
b. C6, C7
c. Humerus
c. C7,C8,T1
d. Ulna
d. C5,C6,C7,C8,T1
44. Lower angle of scapula corresponds to which
36. Partial Claw hand is due to: September 2007 vertebral level: September 2011
a. Radial nerve injury a. D5
b. Ulnar nerve injury b. D7

ANATOMY
c. Median nerve injury c. D9
d. Axillary nerve injury d. D12
37. Structures piercing clavipectoral fascia are all except: THORAX
September 2007 45. The order of neurovascular bundle in the intercostal
a. Cephalic vein space from above downwards is:- March 2005
b. Thoracoacromial artery a. VAN
c. Lateral pectoral nerve b. ANV
d. Basilic vein c. AVN
d. VNA
38. All of the following muscles of the pectoral girdle are
46. Inhaled forgein body usually lodges in the:
innervated by branches from the brachial plexus:
September 2007
except: March 2008
a. Apex of right lung
a. Subclavius b. Lower lobe of right lung
b. Serratus anterior c. Apex of left lung
c. Rhomboid major d. Lower lobe of left lung
d. Trapezius 47. Normal Fluid level in the pericardial cavity:
39. Nerve supply to hypothenar muscles is from : September 2005
a. Ulnar nerve March 2009 a. 50 ml
b. Median nerve b. 100 ml
c. Radial nerve c. 150 ml
d. Musculocutaneous nerve d. 200 ml
48. Right common carotid artery arises from:
40. Structure passing deep to flexor retinaculum at wrist:
September 2005
a. Ulnar nerve September 2009
a. Right axillary artery
b. Median nerve
b. Arch of aorta
c. Radial nerve c. Brachiocephalic artery
d. Ulnar artery d. Left subclavian artery
Self Assessment & Review of FMGE / MCI Screening Examination
4

49. Bifurcation of trachea is at which level: 56. Which of the following does not drain into coronary
September 2006 sinus: March 2009
a. Opposite the disc between the T3-T4 vertebrae a. Anterior cardiac vein
b. Opposite the disc between the T5-T6 vertebrae b. Small cardiac vein
c. Opposite the disc between the T6-T7 vertebrae c. Middle cardiac vein
d. Opposite the disc between the T7-T8 vertebrae d. Great cardiac vein
50. Branches of left coronary artery are all of the 57. True about Thoracic part of sympathetic trunk:
following except: September 2006 September 2007
a. Anterior interventricular branch a. The first five ganglia give preganglionic fibers
b. Left diagonal artery b. The sympathetic trunk has 13 segmentally arranged
c. Left atrial artery ganglia
d. Posterior interventricular branch c. It is the most medially placed structure in the
mediastinum
51. True about arch of aorta are all of the following
d. The first ganglion is often fused with the inferior
except: March 2007
cervical ganglion to form the stellate ganglion
a. Situated behind the lower half of the manubrium
58. Length of esophagus in adults is: September 2005
sterni
a. 25 cm
b. Right common carotid artery arises from the arch of
b. 10 cm
aorta
c. 15 cm
c. It ends at the sternal end of the left second costal
d. 20 cm
cartilage
ANATOMY

d. Begining and end of the arch lies at same level 59. Base of the heart is formed mainly by: September 2008
a. Right atrium
52. Structures passing through diaphragm through aortic
b. Left atrium
hiatus are all except: March 2007
c. Right ventricle
a. Aorta
d. Left ventricle
b. Azygous vein
60. In mid clavicular plane, lower border of lung lies at
c. Thoracic duct
d. Hemiazygous vein level of: September 2008
a. 4th rib
53. If the circumflex artery gives off the posterior
interventricular artery, then the arterial supply is b. 6th rib
called: March 2007 c. 8th rib
a. Right dominance d. 10th rib
b. Left dominance 61. Shape of tracheal cartilage: September 2008
c. Balanced dominanace a. W shaped
d. None of the above b. O shaped
54. Constrictions in esophagus are seen at all the levels c. C shaped
except: March 2007 d. D shaped
a. At the begining of esophagus 62. Arterial supply to SA node is by: March 2009
b. At the site of crossing of esophagus by aortic arch a. Left anterior descending coronary artery
c. Where esophagus pierces the diaphragm b. Posterior interventricular (descending) artery
d. At the point of crossing of thoracic duct c. Left coronary artery
55. Commonest location of diaphragmatic (bochdalek) d. Right coronary artery
hernia in childrens is: March 2007 63. Muscles involved in quiet expiation: September 2009
a. Retrosternal a. Diaphragm
b. Posterior and left b. Intercostal muscles
c. Posterior and right c. Both of the above
d. Central d. None of the above
Anatomy
5

64. Azygous vein drains into: September 2009 c. Inferior Tibiofibular joint
a. Right subcostal vein d. All of the above
b. Superior vena cava 72. Vein used in bypass Surgery: September 2005
c. Braciocephalic a. Great saphenous vein
d. Right ascending lumbar vein b. Short saphenous vein
65. Cardiac dominance is determined by coronary c. Femoral vein
artery: September 2009 d. Brachial vein
a. Supplying circulation to the SA node 73. Muscles taking origin from ischial tuberosity are all
b. Supplying circulation to the inferior portion of the except : March 2006
interventricular septum a. Adductor longus
c. Supplying circulation to the interatrial septum b. Semimembranosus
d. Supplying circulation to the anterior portion of the c. Semitendinosus
interventricular septum d. Adductor magnus
66. SVC syndrome is most commonly associated with: 74. All of the following statements regarding the deep
September 2009 fascia of the thigh are true except: March 2008
a. Mediastinal fibrosis a. It splits in the gluteal region to enclose gluteus
b. Lymphoma maximus muscle
c. Lung cancer b. Iliotibial tract forms a sheath for tensor fasciae latae
d. TB mediastinitis muscle

ANATOMY
67. Arch of aorta begins and ends at which level: c. Medially the fascia is thickened to form iliotibial
a. T2 September 2010 tract
b. T3 d. It is also known as fascia lata
c. T4 75. True regarding relationship of sac in femoral hernia
d. T5 with the pubic tubercle : September 2009
68. Which of these does not form the right border of the a. Above and lateral
heart on X-ray: March 2011 b. Below and lateral
a. Right atrium c. Above and medial
b. Arch of aorta d. Below and medial
c. Superior vena cava 76. Femoral pulsation can be best felt at: March 2011
d. Inferior vena cava a. Below and medial to pubic tubercle
69. Structure passing through the central tendon of b. Near Anterior superior Iliac Spine
diaphragm is: March 2011 c. Mid point of inguinal ligament
a. Esophagus d. Mid-inguinal point
b. Right Phrenic nerve 77. Housemaid’s knee is an inflammation of:
c. Subcostal nerve March 2011
d. Left phrenic nerve a. Lateral patellae bursa
70. Azygos vein drains into: September 2011 b. Semimembranosus bursa
a. Left brachiocephalic vein c. Prepatellar bursa
b. Inferior vena cava d. Suprapatellar bursa
c. Superior vena cava 78. Housemaid knee is an inflammation of:
d. Right brachiocephalic vein September 2011
a. Lateral bursa
INFERIOR EXTREMITY
b. Prepatellar bursa
71. Inversion and eversion occurs at: March 2005
c. Suprapatellar bursa
a. Subtalor joints
d. Anserine bursa
b. Ankle joint
Self Assessment & Review of FMGE / MCI Screening Examination
6

ABDOMEN 87. Blood supply of the uterus is by: March 2007


79. All are branches of Internal Iliac artery except: a. Ovarian artery
March 2005 b. Uetrine artery
a. Ovarian artery c. Both
d. None of the above
b. Superior vesical artery
c. Middle rectal artery 88. Pudendal nerve is related to: March 2007
a. Ischial spine
d. Inferior vesical artery
b. Sacral promontory
80. Which of the following doesn’t prevent prolapse
c. Iliac crest
of uterus: March 2005 d. Ischial tuberosity
a. Perineal body 89. Kidney is supported by all of the following except:
b. Pubocervical ligament March 2007
c. Broad ligament a. Perirenal fat
d. Transverse cervical ligament b. Renal fascia
81. Superficial inguinal lymph nodes drain from all of c. Pararenal fat
the following except: September 2005 d. Fibrous capsule
a. Urethra 90. Uterine artery is a branch of: March 2007
b Anal canal below the pectinate line a. Abdominal aorta
c. Glans penis b. External iliac artery
d. Perineum c. Femoral artery
82. Gerota’s fascia is: September 2005 d. Internal iliac artery
ANATOMY

a. Renal fascia 91. Superficial inguinal ring in the female transmits:


b. Fibrous capsule March 2007
c. Layer of perirenal fat a. Broad ligament of uterus
d. Layer of pararenal fat b. Round ligament of the uterus
83. Superficial inguinal ring is a defect in the: c. Cardinal ligament
d. None of the above
September 2005
92. Regarding kidney, all of the following statements are
a. Internal oblique aponeurosis
true except: September 2007
b. External oblique aponeurosis
a. They lie at the vertebral level T12 to L3
c. Transverse abdominis aponeurosis b. Retroperitoneal organ
d. Internal oblique muscle c. Left kidney is situated lower than the right
84. Pouch of Douglas is between: September 2006 d. Left kidney is located slightly more medial than the
a. Rectum and Sacrum right
b. Uterus and Urinary bladder 93. True about fallopian tubes are all except :
c. Bladder and pubis symphysis September 2007
d. Rectum and Uterus a. Lined by cuboidal epithelium
85. Left testicular vein drains into: September 2006 b. Isthmus is the narrower part of the tube that links to
a. Inferior vena cava the uterus
b. Left renal vein c. Tubal ostium is the point where the tubal canal
c. Portal vein meets the peritoneal cavity
d. Superior vena cava d. Müllerian ducts develops in females into the
86. Which of the following is not supplied by Superior Fallopian tubes
94. Short gastric arteries are branches of: September 2007
mesenteric artery: March 2007
a. Celiac artery
a. Jejunum
b. Splenic artery
b. Appendix
c. Left gastroepiploic artery
c. Ascending colon
d Left gastroepiploic artery
d. Descending colon
Anatomy
7

95. Ovarian artery is a branch of: September 2007 104. Not present at the transpyloric level: March 2009
a. Renal artery a. Neck of pancreas
b. Internal iliac artery b. Fundus of stomach
c. Abdominal part of the aorta c. Left and right colic flexure
d. External iliac artery d. L1 vertebra
96. Commonest position of appendix:
105. Structures lying posterior to spleen are all of the
September 2007, September 2010
following except-: September 2009
a. Pelvic
b. Retrocaecal a. 11th rib
c. Subcaecal b. Left lung
d. Promontoric c. The diaphragm
97. Veins draining into portal vein are all except: d. The stomach
September 2007 106. All of the following veins lack valves except
a. Renal vein September 2009
b. Splenic vein a. Femoral vein
c. Left gastric vein b. Portal vein
d. Superior mesenteric vein c. IVC
98. All of the following structures forms the stomach bed d. Dural venous sinuses
except: March 2008
107. Normal Portal venous pressure is: March 2011
a. Hepatic flexure of colon
a. 5-10 mm Hg
b. The left kidney
b. 10-15 mm Hg

ANATOMY
c. The pancreas
d. Transverse mesocolon c. 15-20 mm Hg
99. Lymphatic drainage of ovary is through: March 2008 d. 20-255 mm Hg
a. Preaortic and para-aortic lymph nodes 108. Renal angle lies between: March 2011
b. Superficial inguinal lymph nodes a. 12th rib and lateral border of sacrospinalis
c. Deep inguinal lymph nodes b. 11th rib and lateral border of quadratus lumborum
d. Internal iliac lymph nodes c. 11th rib and lateral border of sacrospinalis
100. Source of arterial supply to testis: March 2008 d. 12th rib and lateral border of quadratus lumborum
a. Internal pudendal artery 109. All of the following are components of urogenital
b. Deep external pudendal artery diaphragm except: March 2011
c. Superficial external pudendal artery a. Sphincter urethrae
d. Testicular artery
b. Superficial transverse perineii
101. True about uterus are all except: September 2008
c. Deep transverse perineii
a. Normally the uterus is retroverted
d. Superficial fascia of urogenital diaphragm
b. Angle of anteflexion is 125 degree
c. Long axis of uterus corresponds to the axis of pelvic 110. Vaginal fornix is related to which of the following:
inlet September 2011
d. Posterior surface is related to the sigmoid colon a. Morison’s pouch
102. Spleen is supported by all of the following except: b. Pouch of Douglas
September 2008 c. Intersigmoid recess
a. Phrenicocolic ligament d. All of the above
b. Gastrosplenic ligament
HEAD AND NECK
c. Lineorenal ligament
d. Ligamentum teres 111. Muscle which helps to open Eustachian tube while
103. True about ureters are all except: March 2009 sneezing: March 2005, September 2010
a. Constricted at three places a. Tensor veli palatini
b. It is 25cm long b. Levator veli palatini
c. Testicular vessels lie anteriorly to the ureters c. None of the above
d. It is lined by cuboidal epithelium d. Both A and B
Self Assessment & Review of FMGE / MCI Screening Examination
8

112. Superior thyroid artery originates from: 120. Name of the parotid duct: March 2007
September 2005 a. Stensons duct
a. Internal carotid artery b. Nasolacrimal duct
b. External carotid artery c. Whartons duct
c. Facial artery d. None of the above
d. Maxillary artery 121. Parotid duct passes through all the following
113. Recurrent laryngeal nerve supplies all of the structures except: March 2007
following muscles except: September 2005 a. Buccopharyngeal fascia
a. Cricothyroid b. Buccinator
b. Lateral cricoarytenoid c. Buccal fat pad
c. Posterior cricoarytenoid d. Masesster
d. Thyroepiglotticus 122. Thinnest part of scelra is: March 2007
114. True about pharyngeal diverticula are all except: a. At the entrance of optic nerve
September 2005 b. Site of entrance of ciliary nerves
a. Results due to neuromuscular incoordination c. Corneoscleral junction
b. Lies in the anterior wall of pharynx d. At the insertion of recti muscles
c. They are normal in pig 123. Cadaver like position of vocal cords is seen in:
d. Food may get accumulated March 2007
115. Number of parathyroid glands in human: a. Both superior laryngeal nerve palsy
September 2005 b. Both recurrent laryngeal nerve palsy
ANATOMY

a. 4 c. Both external laryngeal nerve palsy


b. 3 d. Both internal laryngeal nerve palsy
c. 2 124. Which of the following extraocular muscles does
d. 5 not arise from annulus: September 2007
116. Nasolacrimal duct opens in: September 2005 a. Inferior rectus
a. The mouth opposite upper 2nd molar b. Medial rectus
b. Middle meatus of nose c. Lateral rectus
c. Superior meatus of nose d. Superior oblique
d. Inferior meatus of nose 125. Elevation of jaw is done by all except:
117. Not a branch of external carotid artery: September 2007
September 2006 a. Temporalis
a. Inferior thyroid artery b. Masseter
b. Facial artery c. Lateral pterygoids
c. Superior thyroid artery d. Medial pterygoids
d. Maxillary artery 126. Structure not passing through the superior orbital
118. False about facial muscles: March 2007 fissure: September 2007
a. Dilates and constrict facial orifices a. Superior ophthalmic vein
b. Supplied by facial nerve b. Trochlear nerve
c. Develops from 3rd pharyngeal arch c. Abducent nerve
d. They develop from mesoderm d. Zygomatic nerve
119. Motor supply to the muscles of the tongue is by: 127. All of the folowing opens into middle meatus
except: September 2007
March 2007
a. Hypoglossal nerve a. Middle ethmoidal air sinuses
b. Maxillary sinus
b. Facial nerve
c. Posterior ethmoid sinuses
c. Lingual nerve
d. Frontal air sinus
d. Glossopharyngeal nerve
Anatomy
9

128. Branches of external carotid artery are all except: b. Facial nerve
September 2007 c. Lingual nerve
a. Maxillary artery d. Glossopharyngeal nerve
b. Ascending pharyngeal artery 136. Parotid duct opens opposite to: March 2009
c. Superior thyroid artery a. Upper 1st molar
d. Ophthalmic artery b. Upper 2nd molar
129. The facial nerve controls all of the following c. Upper 2nd premolar
functions except: March 2008 d. Upper 1st premolar
a. Intensity of the sound reaching the ear 137. Function of superior oblique muscle is: March 2009
b. Lacrimation a. Intortion, adduction and depression
c. Salivation b. Intortion, abduction and elevation
d. Swallowing c. Intortion, abduction and depression
130. All of the following structures are within the d. Extortion, abduction and depression
parotid gland except: March 2008 138. Safety muscle of tongue is: March 2009, September 2010
a. Facial artery a. Styloglossus
b. facial nerve b. Genioglossus
c. External carotid artery c. Palatoglossus
d. Retromandibular vein d. Hyoglossus
131. Which cranial nerve supplies parasympathetic 139. Sensory nerve supply to angle of jaw is through:
secretomotor fibres to the submandibular salivary March 2009

ANATOMY
gland: March 2008 a. Great auricular nerve
a. Vagus b. Buccal branches of facial nerve
b. Trigeminal c. Lesser petrosal nerve
c. Facial d. Auriculotemporal nerve
d. Glossopharyngeal 140. Vocal cord abductors is: March 2009 & 2010;
132. All of the following are contents of the posterior September 2009
triangle of the neck except: March 2008 a. Lateral cricoarytenoid
a. Spinal part of accesory nerve b. Posterior cricoarytenoid
b. Trunks of brachial plexus c. Cricothyroid
c. Internal jugular vein d. Thyroarytenoid
d. Transverse cervical artery 141. The muscle which is not supplied by recurrent
133. Muscle responsible for intorsion of the eye: laryngeal nerve is: September 2009
September 2008 a. Thyroarytenoid
a. Superior oblique b. Posterior cricoarytenoid
b. Superior rectus c. Cricothyroid
d. Lateral cricoarytenoid
c. Both of the above
d. None of the above 142. Facial nerve does not supply which structure of the
face: March 2010
134. Length of the cartilaginous part of external auditory
a. Auricular muscle
canal: September 2008
b. Posterior belly of diagastric muscle
a. 15 mm c. Parotid gland
b. 20 mm d. Submandibular gland
c. 25 mm 143. Muscle involved in congenital torticollis: March 2010
d. 30 mm a. Deltoid
135. The nerve that may get injured during removal of b. Sternocleidomastoid
third molar is : March 2009 c. Digastric
a. Hypoglossal nerve d. Mylohyoid
Self Assessment & Review of FMGE / MCI Screening Examination
10

144. Tendon of which muscle passes through the 151. Scala tympani is supplied by which of the following
pyramid in middle ear: March 2010 nerve: September 2011
a. Incus a. Abducent nerve
b. Stapedius b. Facial nerve
c. Malleus c. Vestibulocochlear nerve
d. Tensor veli palatine d. Glossopharyngeal nerve

145. Which of the following nerve does not supply 152. Oculomotor nerve supplies all muscles of the eye
except: September 2011
submandibular gland: September 2010
a. Inferior oblique
a. Lingual nerve
b. Superior oblique
b. Chorda tympani
c. Superior rectus
c. Sympathetic plexus
d. Inferior rectus
d. Auriculotemporal nerve
146. Which muscle is an abductor of the vocal cords: BRAIN AND SPINAL CORD
March 2011 153. Arterial supply to dura mater is from all of the
a. Oblique Arytenoid following except: September 2005
a. Middle meningeal artery
b. Transverse Arytenoid
b. Internal carotid artery
c. Lateral Thyroarytenoid
c. Ascending pharyngeal artery
d. Posterior Cricoarytenoid d. Basilar artery
147. The palatine tonsil receives its arterial supply from 154. Artery supplying major part of superolateral surface
ANATOMY

all of the following except: March 2011 of brain is: September 2005
a. Facial artery a. Middle cerebral artery
b. Dorsal lingual artery b. Anterior cerebral artery
c. Superior thyroid artery c. Posterior cerebral artery
d. Ascending Palatine artery d. Vertebral artery
155. In adults, spinal cord ends at the lower end of:
148. Sensory nerve supply of the palatine tonsils is by:
September 2005
March 2011
a. L2
a. Greater Palatine nerve
b. L4
b. Trigeminal nerve c. L1
c. Glossopharyngeal nerve d. L3
d. Facial nerve 156. Hypoglossal nerve is: September 2005
149. Which of the following nerves carries taste a. 6th nerve
sensation from posterior one-third of the tongue: b. 7th nerve
March 2011 c. 12th nerve
d. 9th nerve
a. Facial nerve
157. Eye ball movements are controlled by all of the
b. Chorda tympani
following cranial nerves except: September 2005
c. Glossopharyngeal nerve
a. Optic nerve
d. Vagus nerve b. Abducens nerve
150. Internal jugular vein is the continuation of which of c. Trochlear nerve
the following sinus: September 2011 d. Oculomotor nerve
a. Cavernous sinus 158. Largest cranial nerve is: September 2006
b. Sigmoid sinus a. Trochlear
c. Inferior petrosal sinus b. Trigeminal
c. Oculomotor
d. Superior petrosal sinus
d. Vagus
Anatomy
11

159. Not a tributary of cavernous sinus: March 2007 167. Not involved in damage to cerebellum: March 2010
a. Superior petrosal sinus a. Posture
b. Sphenoparietal sinus b. Equiibrium
c. Inferior ophthalmic vein c. Asthenia
d. Superficial middle cerebral veins
d. Tone
160. Nuclei deep within the medial temporal lobes of the
JOINTS
brain: September 2007
a. Amygdala 168. Abduction of the thigh is limited by : September 2009
b. Raphae nucleus a. Ischiofemoral ligaments
c. Dentate nucleus b. Pubofemoral ligaments
d. Red nucleus. c. Iliofemoral ligaments
161. Cross section of midbrain at the level of superior d. Sacroiliac ligament
colliculus shows : September 2009 169. Ligament resisting hyperextension of hip are all
a. Red nucleus and trochlear nerve nucleus except: September 2009
b. Rednucleus and abducent nerve nucleus a. Ischiofemoral ligament
c. Red nucleus and optic nerve nucleus b. Pubofemoral ligament
d. Red nucleus and occulomotor nerve nucleus c. Iliofemoral ligament
162. Spinal part of accesory nerve supplies: March 2009 d. Sacroiliac ligament
a. Pectoralis major
170. Unlocking of knee is done by which muscle:

ANATOMY
b. Pectoralis minor
September 2009
c. Sternocleidomastoid
d. Deltoid a. Adductor magnus

163. Primary auditory area is: March 2009 b. Biceps femoris


a. Inferior temporal gyrus c. Popliteus
b. Occiptal cortex d. Sartorius
c. Superior temporal gyrus MISCELLANEOUS
d. Frontal cortex 171. Auricle of the ear is made of: March 2007
164. A lesion involving upper motor neuron of facial a. Hyaline cartilage
nerve manifests as: March 2009
b. Fibrocartilage
a. Upper half of the face is affected, lower half normal
c. Elastic cartilage
b. Left half of the face
d. None of the above
c. Right half of the face
d. Upper half of the face normal, lower half affected 172. Elastic cartilage is absent in: March 2011
165. Nucleus ambiguus is not associated with which a. Epiglottis
cranial nerve: March 2010 b. Trachea
a. X c. External auditory canal
b. XI d. Auricle of the ear
c. IX 173. Arterio-venous anastomosis is not seen in:
d. XII March 2011
166. In neonates, spinal cord ends at: March 2010 a. Sympathetic ganglia
a. L1 b. Tip of Nose
b. L2 c. Finger tip
c. L3 d. Kidney
d. L4
ANSWERS WITH EXPLANATIONS
EMBRYOLOGY
1. The umbilical cord is surrounded by the fetal membrane, amnion, and contains Wharton‖s jelly. Embedded in this
jelly are the:
i. Remains of the vitellointestinal duct and the allantois
ii. Two umbilical arteries
iii. One umbilical vein

The vein is a larger thin-walled vessel and is located at the 12 o‖clock position when facing the umbilicus; the two
arteries, which lie adjacent to one another and are located at the 4 and 8 o‖clock positions when facing the umbilicus,
are smaller and thick walled.

Ans. D: 2 arteries and 1 vein


Ref.: Langman‖s medical embryology, 9th ed., page-147

2. The ductus arteriosus represents the distal portion of the sixth left aortic arch and connects the left pulmonary artery
to the beginning of the descending aorta.
• During fetal life, blood passes through it from the pulmonary artery to the aorta, thus bypassing the lungs. After
birth, it normally constricts, later closes, and becomes the ligamentum arteriosum.
• A persistent patent ductus arteriosus results in high-pressure aortic blood passing into the pulmonary artery,
which raises the pressure in the pulmonary circulation.
• A patent ductus arteriosus is life threatening and should be ligated and divided surgically.

Ans. A: Ductus arteriosus


Ref.: BDC 4th ed., vol.1, page-261, Clinical Anatomy-Snell, 8th ed., page-126

3. The fourth aortic arch persists on both sides, but its ultimate fate is different on the right and left sides.
• On the left it forms part of the arch of the aorta, between the left common carotid and the left subclavian arteries.
• On the right it forms the most proximal segment of the right subclavian artery, the distal part of which is formed by
a portion of the right dorsal aorta and the seventh intersegmental artery

Ans. C: Left fourth aortic arch artery


Ref.: BDC 4th ed., vol.1, page-263, Langman‖s medical embryology, 9th ed., page-256

4. • The eyes begin to develop as a pair of outpocketings that will become the optic vesicles on each side of the
forebrain at the end of the fourth week of development.

• The optic vesicles contact the surface ectoderm and induce lens formation.

• When the optic vesicle begins to invaginate to form the pigment and neural layers of the retina, the lens placode
invaginates to form the lens vesicle.

Ans. B: Surface ectoderm


Ref.: BDC 4th ed., vol.3, page-277, Langman‖s medical embryology, 9th ed., page-426
Anatomy 13

5. Early in development, the embryonic mesoderm becomes differentiated into three distinct regions: paraxial mesoderm,
intermediate mesoderm, and lateral mesoderm.
The paraxial mesoderm gets divided into somites which gets differentiated into a ventromedial part (the sclerotome)
and a dorsolateral part (the dermatomyotome).
The dermatomyotome now further differentiates into the myotome and the dermatome.

The mesenchymal cells of the sclerotome rapidly divide and migrate medially during the fourth week of development
and surround the notochord.

The caudal half of each sclerotome now fuses with the cephalic half of the immediately succeeding sclerotome to form
the mesenchymal vertebral body.
Each vertebral body is thus an intersegmental structure. The notochord degenerates completely in the region of the
vertebral body, but in the intervertebral region, it enlarges to form the nucleus pulposus of the intervertebral discs.

The surrounding fibrocartilage, the anulus fibrosus, of the intervertebral disc is derived from sclerotomic mesenchyme
situated between adjacent vertebral bodies.

Ans. B: Nucleus pulposus


Ref.: Clinical Anatomy-Snell, 8th ed., page-875

6. With differential growth of the dorsal bladder wall, the ureters come to open through the lateral angles of the bladder,

ANATOMY
and the mesonephric ducts open close together in what will be the urethra.

That part of the dorsal bladder wall marked off by the openings of these four ducts forms the trigone of the bladder.

Thus, lining of the bladder over the trigone is mesodermal in origin;

The smooth muscle of the bladder wall is derived from the splanchnopleuric mesoderm.

The apex of the bladder is continuous with the allantois, which now becomes obliterated and forms a fibrous core, the
urachus.

The urachus persists throughout life as a ligament that runs from the apex of the bladder to the umbilicus and is called
the median umbilical ligament

Ans. A: Mesoderm
Ref.: BDC 4th ed., vol.2, page-351, Clinical Anatomy-Snell, 8th ed., page-357

7. Fossa ovalis and anulus ovalis lie on the atrial septum, which separates the right atrium from the left atrium.

The fossa ovalis is a shallow depression, which is the site of the foramen ovale in the fetus. The anulus ovalis forms the
upper margin of the fossa. The floor of the fossa represents the persistent septum primum of the heart of the
embryo, and the anulus is formed from the lower edge of the septum secundum

Ans. B: Septum primum


Ref.: BDC 4th ed., vol.1, page-244, Clinical Anatomy-Snell, 8th ed., page-107

8. The eardrum forms from the joining of the expanding first pharyngeal pouch and groove. Around day 30 of gestation,
the endoderm-lined first expands to form the tympanic cavity, which subsequently envelops the inner ear ossicles.
Self Assessment & Review of FMGE / MCI Screening Examination
14

Simultaneously, the first pharyngeal groove, which is lined with ectoderm, expands to form the developing external
auditory meatus. Separated by a thin layer of splanchnic mesoderm, the tympanic cavity and external auditory meatus
join to form the tympanic membrane.
As a result, the tympanic membrane is derived from all three germ layers.

Ans. B: Tympanic membrane


Ref.: BDC 4th ed., vol.3, page-266

9. Diverticula are classified as true and false. True diverticula are composed of all layers of the intestinal wall, whereas

false diverticula are formed from the herniation of the mucosal and submucosal layers.

Diverticula can be classified as intraluminal or extraluminal.


a. Intraluminal diverticula and Meckel diverticulum are congenital.
b. Extraluminal diverticula may be found in various anatomic locations and are referred to as duodenal, jejunal,
ileal, or jejunoileal diverticula.
Meckel‖s diverticulum is a congenital anomaly representing a persistent portion of the vitellointestinal duct. It occurs
in 2% of patients, is located about 2 ft (61 cm) from the ileocolic junction, and is about 2 inch. (5 cm) long.
It can become ulcerated or cause intestinal obstruction.
A Meckel’s diverticulum, a true congenital diverticulum, is a small bulge in the small intestine present at birth. It is a
vestigial remnant of the omphalomesenteric duct (also called the vitelline duct or yolk stalk), and is the most frequent
ANATOMY

malformation of the gastrointestinal tract. It is present in approximately 2% of the population, with males more
frequently experiencing symptoms.
A memory aid is the rule of 2‖s: 2% (of the population) - 2 feet (from the ileocecal valve) - 2 inches (in length) - 2% are
symptomatic, there are 2 types of common ectopic tissue (gastric and pancreatic), the most common age at clinical
presentation is 2, and males are 2 times as likely to be affected.

Ans. B: Meckel’s diverticulum


Ref.: BDC 4th ed., vol.2, page-252,251. Clinical Anatomy-Snell, 8th ed., page-182

10. The boundaries of anatomical snuff box:


i. The medial border of the snuffbox is the tendon of the extensor pollicis longus.
ii. The lateral border is a pair of parallel and intimate tendons, of the extensor pollicis brevis and the abductor pollicis
longus. (Accordingly, the anatomical snuffbox is most visible, having a more pronounced concavity, during
thumb extension.)
iii. The proximal border is formed by the styloid process of the radius
iv. The distal border is formed by the approximate apex of the schematic snuffbox isosceles triangle.
v. The floor of the snuffbox varies depending on the position of the wrist, but both the trapezium and primarily the
scaphoid can be palpated
Deep to the tendons which form the borders of the anatomical snuff box lies the radial artery, which passes
through the anatomical snuffbox on its course from the normal radial pulse detecting area, to the proximal space in
between the first and second metacarpals to contribute to the superficial and deep palmar arches.

Ans. C: Radial artery


Ref.: Snell‖s Clinical Anatomy, 7th ed., p-533
Anatomy 15

11. The constituents of the umbilical cord when fully developed are covering epithelium, wharton’s jelly, blood vessels, remnant of the
umbilical vesicle (yolk sac) and its vitelline duct, allantois and obliterated extraembryonic coelom

Umbilical cord:
 The umbilical cord develops from and contains remnants of the yolk sac and allantois.
 It forms by the fifth week of fetal development, replacing the yolk sac as the source of nutrients for the fetus.
 The length of the umbilical cord is approximately equal to the crown-rump length of the fetus throughout
pregnancy.
 The umbilical cord in a full term neonate is usually about 50 centimeters (20 in) long and about 2 centimeters
(0.75 in) in diameter.
 The umbilical cord is composed of Wharton's jelly, a gelatinous substance made largely from
mucopolysaccharides.
 It contains one vein, which carries oxygenated, nutrient-rich blood to the fetus, and two arteries that carry
deoxygenated, nutrient-depleted blood away.
 Occasionally, only two vessels (one vein and one artery) are present in the umbilical cord.
 The blood flow through the umbilical cord is approximately 35 ml / min at 20 weeks, and 240 ml / min at 40
weeks of gestation.
 Adapted to the weight of the fetus, this corresponds to 115 ml / min / kg at 20 weeks and 64 ml / min / kg at 40

ANATOMY
weeks

Cloacal duct
 It is a small communication between the 2 portions of the hindgut.
 Down growth of the urorectal septum is believed to close this duct by 7 weeks' gestation.

Ans. C: Cloacal duct


Ref.: Dutta‖s Obstetrics, 7th ed., p-40

12. Umbilical vein carries oxygenated blood to embryo

Ans. B: Carry oxygenated blood towards the fetus


Ref.: IB Singh‖s Embryology, 7th ed., p-265

13. After birth, the left umbilical vein are obliterated and forms the ligamentum teres hepatis

Ligamentum teres

 It is the obliterated fibrous remnant of the left umbilical vein of the fetus.
 It originates at the umbilicus.
 It passes superiorly in the free margin of the falciform ligament.
 From the inferior margin of the liver, it may join the left branch of the portal vein or it may be in continuity with
the ligamentum venosum
Self Assessment & Review of FMGE / MCI Screening Examination
16

Other fetal remnants

 Umbilical arteries forms medial umbilical ligament


 Ductus venosus forms ligamnetum venosusm
 Ductus arteriosus forms ligamnetum arteriosum

Ans. B: Remnant of umbilical vein


Ref.: IB Singh‖s Embryology, 7th ed., p-265

14. Complete anatomical obliteration by proliferation of the intima is thought to take 1-3 months.

Patent ductus arteriosus (PDA)


 Failure of a child's DA to close after birth results in a condition called patent ductus arteriosus and the generation
of a left-to-right shunt.
 If left uncorrected, patency leads to pulmonary hypertension and possibly congestive heart failure and cardiac
arrhythmias.
 Prostaglandins are responsible for maintaining the ductus arteriosus.
 Closure may be induced with a drug class known as NSAIDs such as indomethacin or ibuprofen because these
drugs inhibit prostaglandin synthesis.
 A patent ductus arteriosus affects around 40% of infants with Down syndrome (DS).
ANATOMY

Changes in circulation after birth


 The ductus arteriosus is occluded, so that all blood from the right ventricle now goes to the lungs, where it is
oxygenated.
 Initial closure of the DA is caused by contrcation of the muscle in the vessel wall
 Later in 1-3 months intima proliferation obliterates the lumen
So looking at the choices, ―D‖ option seems the best answer

Ans. D: 30 day
Ref.: IB Singh‖s Embryology, 6th ed., p-259

15. In 2-4% of people, a small portion of the vitelline duct persists, forming an outpocketing of the ileum, Meckel’s/ ileal diverticulum
Sometimes both ends of the vitelline duct transforms into fibrous cords, and the middle portion forms a large cyst, an
enterocystoma/ vitelline cyst
Vitelline duct, ay remain patent over its entire length, forming a direct communication between the umbilicus and the intestinal
tract known as umbilical fistula/ vitelline fistula

Vitello-intestinal duct or omphalo-mesenteric duct connects the midgut to the yolk sac during early embryonic life
and gets obliterated and disappears during fifth to sixth week of intrauterine life. If the remnants persists then
following abnormalities can occur:
 Umbilical Polyp: Umbilical polyp occur because of persistance of small portion of vitello-intestinal duct
epithelium at the base of umbilicus.
 Umbilical Sinus: When umbilical portion of vitello-intestinal duct remains patent, an umbilical sinus forms.
Anatomy 17

 Fibrous remnant of vitello-intestinal duct: Whole vitello-intestinal duct become fibrous strand but does not
disappear.
 Meckel's/ Ileal Diverticulum: The ileal portion of vitello-intestinal duct remains patent and form a diverticulum,
called as Meckel's diverticulum.
 Patent Vitello-intestinal duct: When whole vitello-intestinal duct remains patent cause fistulous connection
between umbilicus and ileum (umbilical fecal fistula)

Mesenteric cyst
 Mesenteric cyst are thought to represent benign proliferations of ectopic lymphatics that lack communication with
the normal lymphatic system.

 Cysts are thought to arise from lymphatic spaces associated with the embryonic retroperitoneal lymph sac,
making them analogous to cystic hygromas, which arise in the neck in association with the jugular lymph sac.

 Another proposed etiology is lymphatic obstruction

Mesenteric cysts can occur anywhere in the mesentery of the gastrointestinal tract from the duodenum to the
rectum, and they may extend from the base of the mesentery into the retroperitoneum

Ans. D: Mesenteric cyst


Ref.: Langman‖s Embryology, 11th ed., p-228

ANATOMY
16. In the human, trophoblastic cells over the embryoblast pole begin to penetrate between the epithelial cells of the
uterine mucosa on about the 6th day

Implantation: The second week of human development is concerned with the process of implantation and the
differentiation of the blastocyst into early embryonic and placental forming structures.

 Implantation commences about day 6

 Adplantation - begins with initial adhesion to the uterine epithelium (blastocyst then slows in motility, "rolls" on
surface, aligns with the inner cell mass closest to the epithelium and stops)

 Implantation - migration of the blastocyst into the uterine epithelium, process complete by about day 9

 Coagulation plug - left where the blastocyst has entered the uterine wall day 12

 Normal Implantation Sites - in uterine wall superior, posterior, lateral

Ans. D: 6
Ref.: Langman‖s Embryology, 11th ed., p-41

17. Although initially only slightly motile, spermatozoa obtains full motility in the epididymis

Development of the sperm


 Spermatogenesis is the process of spermatagonia (diploid) mature into spermatozoa (haploid).
 Spermioogenesis is a part of spermatogenesis where round spermatids mature into the mature spermatozoa
form.
Self Assessment & Review of FMGE / MCI Screening Examination
18

 Spermatozoa acquire some motility only after passing through the epididymis
 The secretions of the epididymis, seminal vesicle and the prostate have a stimulating effect on sperm motility, but
the sperm becomes fully motile only after ejaculation
 Continuously throughout life occurs in the seminiferous tubules in the male gonad-testis.
 At puberty spermatagonia activate and proliferate (mitosis).
 About 48 days from entering meiosis until morphologically mature spermatozoa
 About 64 days to complete spermatogenesis, depending reproduction time of spermatogonia
 Follicle stimulating hormone (FSH) - stimulates the spermatogenic epithelium
 Luteinizing-hormone (LH) - stimulates testosterone production by Leydig cells
Stages of spermatozoa development
 Spermatogonia - are the first cells of spermatogenesis
 Primary spermatocytes - large, enter the prophase of the first meiotic division
 Secondary spermatocytes - small, complete the second meiotic division
 Spermatid - immature spermatozoa
 Spermatozoa - differentiated gamete

Ans. D: Epididymis
ANATOMY

Ref.: Langman‖s Embryology, 11th ed., p-30; Guyton‖s Physiology, 10th ed., p-918

18. The genital tubercle elongates only slightly and forms the clitoris
Phallic tubercle or genital tubercle
 Present in the development of the urinary and reproductive organs.
 It forms in the ventral, caudal region of mammalian embryos of both sexes and eventually develops into a
phallus.
 In the human fetus the genital tubercle develops around week 4 of gestation and by week 9 becomes recognizably
either a clitoris or penis.
 This should not be confused with the sinus tubercle which is a proliferation of endoderm induced by
paramesonephic ducts.
Even after the phallus is developed, the term genital tubercle remains, but only as the terminal end of it, which
develops into either the glans penis or the glans clitoridis.

Ans. A: Genital tubercle


Ref.: IB Singh‖s Embryology, 6th ed., p-279

HISTOLOGY
19. Ans. A: Nonkeratinized Stratified Squamous epithelium
Ref.: BDC 4th ed., vol.2, page-365

20. Brunner glands (or Pancreal glands/duodenal glands) are compound tubular submucosal glands found in that
portion of the duodenum which is above the hepatopancreatic sphincter.
Anatomy 19

The main function of these glands is to produce a mucus-rich alkaline secretion (containing bicarbonate).

Ans. B: Duodenum
Ref.: BDC 4th ed., vol.2, page-251, IB Singh histology-5th ed-page-244

21. Layers of the retina, from the outside inwards


i. Retinal Pigment Epithelium
ii. Layer of rods and cones
iii. External (Outer) limiting membrane
iv. Outer nuclear layer
v. Outer plexiform layer
vi. Inner nuclear layer
vii. Inner plexiform layer
viii. Ganglion cell layer
ix. Layer of optic nerve fibres
x. Internal (Inner) limiting membrane
The neural retina meets the optic nerve at the optic disc (or optic papilla), which is devoid of photoreceptor cells,
forming a blind spot in the visual field.
The fovea, lateral to the optic disc, is the area of greatest visual acuity, and is surrounded by a yellow pigmented zone

ANATOMY
called the fovea lutea.
Bowman’s membrane is present in cornea

Ans. C: Bowman’s membrane


Ref.: BDC 4th ed., vol.3, page-275,271, IB Singh histology-5th ed-page-328

22. Cardiac muscle consists of striated muscle fibers that branch and unite with each other. It forms the myocardium of
the heart.
Its fibers tend to be arranged in whorls and spirals, and they have the property of spontaneous and rhythmic
contraction.
Specialized cardiac muscle fibers form the conducting system of the heart.
Cardiac muscle is supplied by autonomic nerve fibers that terminate in the nodes of the conducting system and in the
myocardium.

Ans. C: Cardiac muscle cells are linear and longitudinal


Ref.: IB Singh histology-5th ed-page-132, Clinical Anatomy-Snell, 8th ed., page-12

23. Other sites with the typical cuboidal epithelium are:


i. Surface of ovary
ii. Choroid plexus
iii. Inner surface of lens
iv. Pigment cell layer of retina

Ans. D: Thyroid gland


Ref.: IB Singh histology-5th ed-page-48
Self Assessment & Review of FMGE / MCI Screening Examination
20

24. Hasal’s corpuscles /thymic corpuscles are structures found in the medulla of the human thymus, formed from type

VI epithelial reticular cells.


They are named for Arthur Hill Hassall, who discovered them in 1849.

Ans. A: Thymus
Ref.: IB Singh histology-5th ed-page-194

25. Blood-Testis Barrier a barrier separating the blood from the seminiferous tubules, consisting of special junctional
complexes between adjacent Sertoli cells near the base of the seminiferous epithelium.

Other barriers in the body:

i. Placental barrier term sometimes used for the placental membrane, because it prevents the passage of some
materials between the maternal and fetal blood.

ii. Blood-aqueous barrier the physiologic mechanism that prevents exchange of materials between the chambers of
the eye and the blood.

iii. Blood-brain barrier, blood-cerebral barrier the selective barrier separating the blood from the parenchyma of the
central nervous system. Abbreviated BBB

Ans. B: Sertoli cells


ANATOMY

Ref.: IB Singh histology-5th ed-page-286

26. Continuos capillaries


Location: muscle, lung, central nervous system and skin.
Composition: endothelial cells contain numerous pinocytotic vesicles, pericytes are enclosed by a basement
membrane.
Fenestrated Capillaries
Location: endocrine glands, sites of fluid and metabolite absorption: renal corpuscles, intestinal tract, and
gallbladder.
Composition: endothelial cells contain fenestrations, 80-100 nm in diameter, those provide channels across the
capillary wall, pericytes are enclosed by a basement membrane.
Discontinuous capillaries (sinusoid)
Location: liver, spleen, and bone marrow.
Composition: endothelial cells with unusually wide gaps between them, partial or total absence of basement
membrane.
Ans. D: Muscle
Ref.: IB Singh histology-5th ed-page-180

27. The goblet cells secrete mucus, a viscous fluid composed primarily of highly glycosylated proteins called mucins.

Goblet cells are found scattered among other cells in the epithelium of many organs, especially in the intestinal and
respiratory tracts.
Anatomy 21

They are present in trachea, bronchus and larger bronchioles in respiratory tract, small intestines, the colon and
conjunctiva in the upper eye lid

In some areas, their numbers are rather small relative to other cell types, while in tissues such as the colon, they are
much more abundant

Ans. D: Esophagus
Ref.: IB Singh histology-5th ed-page-47, 246

28. Peyer's patches are observable as elongated thickenings of the intestinal epithelium measuring a few centimeters in
length. About 30 are found in humans.
Microscopically, Peyer’s patches appear as oval or round lymphoid follicles (similar to lymph nodes) located in the
lamina propria layer of the mucosa and extending into the submucosa of the ileum

Ans. C: Ileum
Ref.: DiFiore‖s Histology, 11th ed.,p-300

29. In the white pulp, the T-cells areas surround the central arteries, forming the periarteriolar sheath (PALS)
White pulp
 The white pulp consists of lymphatic tissue surrounding arteries and nodules also associated with arteries.
 The nodules can be distinguished because of the arteries (central arteries).

ANATOMY
 The lymphatic tissue immediately surrounding the central artery is known as the periarterial lymphatic sheath
(PALS) and is composed of T-lymphocytes.
 The more peripheral part of the nodules is known as the peripheral white pulp (PWP) and consists of aggregates of
B-lymphocytes.
Red pulp
 The red pulp is like a sponge composed of cords of cells (splenic cords) and splenic sinusoids (venous sinusoids).
 The splenic cords (Billroth cords) are composed of :
o Reticular cells and fibers
o Fixed and wandering macrophages
o Lymphocytes
o Plasma cells
Blood cells (erythrocytes, granulocytes) and pl

Ans. C: Kidney
Ref.: IB Singh histology-5th ed-page-132, Clinical Anatomy-Snell, 8th ed., page-12

30. The lining epithelium of uterus is columnar


Before menarche (the age of menstruation) the cells are ciliated, but thereafter most of the cells may not have cilia
Ciliated columnar epithelium
 These cells are rectangular in shape and have between 200 to 300 hair-like protrusions called cilia T
 The mitochondria are found toward the apical region of the cell while the cell nuceli are found towards the base
and are often elongated.
Self Assessment & Review of FMGE / MCI Screening Examination
22

 Cells are interconnected via desmosomses and tight junctions, creating a semipermeable membrane that is more
selective that membrane found in other types of cell.
 Ciliated columnar epithelial cells are found mainly in the tracheal and bronchial regions of the pulmonary system
and also in the fallopian tubes of the female reproductive system.

Ans. A: Ciliated columnar epithelium


Ref.: IB Singh histology-6th ed-page-312,

SUPERIOR EXTREMITY

31. BRANCHES OF THE BRACHIAL PLEXUS


There are a total of 17 branches arising from the brachial plexus that are destined to supply the upper limb of the
seventeen branches of the brachial plexus; three of the branches arise from the root, one from the trunk, three from the
lateral cord, five from the medial cord and five from the posterior cord.
Branches from the roots
i. Long thoracic nerve of bell (C5, C6, C7).
ii. Dorsal scapular nerve (C5).
Branches from the trunk
i. Suprascapular Nerve (C5, C6)
ANATOMY

ii. Nerve to subclavius (C5, C6)


Branches from the lateral cord
i. Lateral pectoral nerve. (C5, C6).
ii. Musculocutaneous – (C5, C6, C7)
iii. Lateral root of median nerve (C5, C6, C7).
Branches from the medial cord
i. Medial pectoral nerve (C8, T1)
ii. Medial cutaneous nerve of arm (C8,T1)
iii. Medial cutaneous nerve of forearm (C8,T1)
iv. Ulnar nerve(C7, C8, T1)
v. Medial root of median nerve (C8,T1)
Branches from the posterior cord
i. Axillary nerve (C5, C6)
ii. Upper subscapular nerve (C5,C6)
iii. Thoracodorsal nerve (C6, C7, C8).
iv. Lower subscapular nerve (C5, C6)
v. Radial nerve (C5-T1).

Ans. D: C5, C6, C7, C8, T1


Ref.: BDC 4th ed., vol.1, page-52, Clinical Anatomy-Snell, 8th ed., page-450

32. • The ulnar artery is the larger of the two terminal branches of the brachial artery.
• It begins in the cubital fossa at the level of the neck of the radius.
Anatomy 23

• It descends through the anterior compartment of the forearm and enters the palm in front of the flexor retinaculum
in company with the ulnar nerve.
• It ends by forming the superficial palmar arch, often anastomosing with the superficial palmar branch of the radial
artery.
• In the upper part of its course, the ulnar artery lies deep to most of the flexor muscles.
• Below, it becomes superficial and lies between the tendons of the flexor carpi ulnaris and the tendons of the flexor
digitorum superficialis.
• In front of the flexor retinaculum, it lies just lateral to the pisiform bone.
Branches

i. Muscular branches to neighboring muscles


ii. Recurrent branches that take part in the arterial anastomosis around the elbow joint
ii. Branches that take part in the arterial anastomosis around the wrist joint
iv. The common interosseous artery, which arises from the upper part of the ulnar artery and after a brief course
divides into the anterior and posterior interosseous arteries.

Ans. C: Ulnar artery


Ref.: BDC 4th ed., vol.1, page-107, Clinical Anatomy-Snell, 8th ed., page-486

33. Pectoralis major arises from the anterior surface of the sternal half of the clavicle; from breadth of the half of the

ANATOMY
anterior surface of the sternum, as low down as the attachment of the cartilage of the sixth or seventh rib; from the
cartilages of all the true ribs, with the exception, frequently, of the first or seventh and from the aponeurosis of the
abdominal external oblique muscle.

From this extensive origin the fibers converge in a flat tendon, about 5 cm in breadth, which is inserted into the lateral
lip of the bicipital groove of the humerus.

Ans. A: Lateral lip of bicipital groove of humerus


Ref.: BDC 4th ed., vol.1, page-45, Clinical Anatomy-Snell, 8th ed., page-441

34. The median nerve controls the coarse movements of the hands, as it supplies most of the long muscles of the front of
the forearm and therefore called the ‘labourer’s nerve’.
The median nerve is formed from parts of the medial and lateral cords of the brachial plexus
The median nerve is the only nerve that passes through the carpal tunnel.
Innervation

i.Upper Arm
No motor innervation.
ii. Forearm
It innervates most of the flexors in the forearm except flexor carpi ulnaris and the medial two digits of flexor
digitorum profundus, which are supplied by the ulnar nerve.
Unbranched, the median nerve supplies the following muscles:
i. Pronator teres
ii. Flexor carpi radialis
iii. Palmaris longus
iv. Flexor digitorum superficialis muscle.
Self Assessment & Review of FMGE / MCI Screening Examination
24

The anterior interosseus branch supplies the following muscles:


i. Lateral (radial) half of flexor digitorum profundus muscle
ii. Flexor pollicics longus muscle
iii. Pronator quadratus
iv. Hand.

In the hand, the median nerve supplies motor innervation to the 1st and 2 nd lumbricals and the muscles of the thenar
eminence of the hand by a recurrent thenar branch.
The rest of the intrinsic muscles of the hand are supplied by the ulnar nerve.
Injury
a. Injury of this nerve at a level above elbow joint results in loss of pronation and a decrease in flexion of the hand at
the wrist joint.
b. In the hand, thenar muscle are paralysed and atrophy with in time. Opposition and flexion movements of thumb
are lost, and thumb and index finger are arrested in adduction and hyperextension position. This appearance is
referred as ape hand deformity.
In addition, in palmar side of the hand sensation of lateral part of hand, first three fingers and lateral half of the
fourth finger and in dorsal side sensation of distal S! portion of first three fingers and lateral half of distal S!
portion of fourth finger is lost.

Ans. A: Median nerve


ANATOMY

Ref.: BDC 4th ed., vol.1, page-110

35. Erb-Duchenne Palsy


Upper lesions of the brachial plexus are injuries resulting from excessive displacement of the head to the opposite side
and depression of the shoulder on the same side. This causes excessive traction or even tearing of C5 and C6 roots of
the plexus.
It occurs in infants during a difficult delivery or in adults after a blow to or fall on the shoulder.
The suprascapular nerve, the nerve to the subclavius, and the musculocutaneous and axillary nerves all possess nerve
fibers derived from C5 and C6 roots and will therefore be functionless.
The following muscles will consequently be paralyzed: the supraspinatus (abductor of the shoulder) and infraspinatus
(lateral rotator of the shoulder); the subclavius (depresses the clavicle); the biceps brachii (supinator of the forearm,
flexor of the elbow, weak flexor of the shoulder) and the greater part of the brachialis (flexor of the elbow) and the
coracobrachialis (flexes the shoulder); and the deltoid (abductor of the shoulder) and the teres minor (lateral rotator of
the shoulder).
Thus, the limb will hang limply by the side, medially rotated by the unopposed sternocostal part of the pectoralis
major; the forearm will be pronated because of loss of the action of the biceps.
The position of the upper limb in this condition has been likened to that of a porter or waiter hinting for a tip.
In addition, there will be a loss of sensation down the lateral side of the arm.
Treatment
The three most common treatments from Erb‖s Palsy are: Nerve transfers (usually from the opposite leg),
sub scapularis releases and latissimus dorsi tendon transfers.

Ans. A: C5, C6
Ref.: BDC 4th ed., vol.1, page-53
Anatomy 25

36. The metacarpophalangeal joints become hyperextended because of the paralysis of the lumbrical and interosseous
muscles, which normally flex these joints.
Because the first and second lumbricals are not paralyzed (they are supplied by the median nerve), the hyperextension
of the metacarpophalangeal joints is most prominent in the fourth and fifth fingers.
The interphalangeal joints are flexed, owing again to the paralysis of the lumbrical and interosseous muscles, which
normally extend these joints through the extensor expansion.
The flexion deformity at the interphalangeal joints of the fourth and fifth fingers is obvious because the first and
second lumbrical muscles of the index and middle fingers are not paralyzed.
In long-standing cases the hand will show hollowing between the metacarpal bones caused by wasting of the dorsal
interosseous muscles
True/complete claw hand involving all the fingers is produced by a combined lesion of ulnar and median nerve

Ans. B: Ulnar nerve injury


Ref.: BDC 4th ed., vol.1, page-124, Clinical Anatomy-Snell, 8th ed., page-536

37. The clavipectoral fascia is a strong sheet of connective tissue that is attached above to the clavicle.
Below, it splits to enclose the pectoralis minor muscle and then continues downward as the suspensory ligament of the
axilla and joins the fascial floor of the armpit.
The coracoclavicular fascia is pierced by the cephalic vein, thoracoacromial artery and vein, lymphatics pasing from

ANATOMY
the breast and pectoral region to the apical group of axillary lymph nodes and lateral pectoral nerve.

Ans. D: Basilic vein


Ref.: BDC 4th ed., vol.1, page-46, Clinical Anatomy-Snell, 8th ed., page-444

38. Trapezius is supplied by spinal part of cranial nerve XI

Ans. D: Trapezius
Ref.: BDC 4th ed., vol.1, page-52, Clinical Anatomy-Snell, 8th ed., pge-447

39. The muscles of hypothenar eminence are:


i. Opponens digiti minimi
ii. Flexor digiti minimi
iii. Abductor digiti minimi
iv. Palmar brevis
The intrinsic muscles of hand can be remembered using the mnemonic, “A OF A OF A (P)” for, Abductor pollicis
longus, Opponens pollicis, Flexor pollicis brevis, Adductor pollicis (thenar muslces) and Opponens digiti minimi,
Flexor digiti minimi, Abductor digiti minimi and Palmar brevis (Hypothenar muscles)
They are all supplied by the deep branch of the ulnar nerve.

Ans. A: Ulnar nerve


Ref.: BDC 4th ed., vol.1, page-117, Clinical Anatomy-Snell, 8th ed., page-506

40. The flexor retinaculum stretches across the front of the wrist and converts the concave anterior surface of the hand into
an osteofascial tunnel, the carpal tunnel, for the passage of:
Self Assessment & Review of FMGE / MCI Screening Examination
26

i. The median nerve

ii. Flexor tendons of the thumb (flexor pollicis longus and fingers) (flexor digitorum superficialis and profundus).

iii. Radial and the ulnar bursa

It is attached medially to the pisiform bone and the hook of the hamate and laterally to the tubercle of the scaphoid
and the trapezium bones.

The attachment to the trapezium consists of superficial and deep parts and forms a synovial-lined tunnel for passage
of the tendon of the flexor carpi radialis.

The lower border is attached to the palmar aponeurosis.

Ans. B: Median nerve


Ref.: BDC 4th ed., vol.1, page-113, Clinical Anatomy-Snell, 8th ed., page-484

41. Coracoclavicular ligaments:


- Stronger, vertically directed contains conoid and trapezoid ligaments help to control vertical stability

Coracoclavicular ligament are suspensory ligaments of upper limb.


i. Conoid:
- Is the most important ligament for support of the joint against significant injuries and superior displacement;
- Cone shaped which extends between the conoid tubercle on the posterior clavicle and the base of the coracoid.
ANATOMY

ii. Trapezoid:
• Resists AC joint compression;
• Begins anteriorly and laterally to the conoid ligament on the clavicle and inserts on the coracoid process.

Ans. B: Coracoclavicular ligament


Ref.: Internet resources

42. Anatomy of radial nerve


I. Formed by: Axons from
a. Roots: C5 to T1
b. Brachial plexus
i. Trunks: Superior, Medial and Inferior
ii. Cord: Posterior
II. Axons pass through
a. Spiral groove of humerus
b. Fibrous arch attachment of triceps to humerus
c. Lateral intermuscular septum below deltoid insertion
d. Arcade of Frohse: Above supinator and below elbow
III. Branches
A. Above elbow
i. Above spiral groove (humerus)
• Cutaneous nerves to arm: Posterior; Lower lateral
• Cutaneous nerve to forearm: Posterior
• Triceps brachii: 3 to 5 branches
• Anconeus
Anatomy 27

ii. Below spiral groove


• Brachioradialis: Distal to lateral intermuscular septum
• Extensor carpi radialis longus and brevis
• Supply lateral and dorsolateral arm and forearm
B. At or below elbow
a. Above arcade of Frohse (Supinator): Final bifurcation
• Superficial radial nerve: Sensory dorsolateral hand and First 3 digits
• Posterior interosseus nerve (deep branch): Extensor carpi radialis brevis & supinator
b. Passes through arcade of Frohse
• Arcade of Frohse location: Between 2 heads of supinator muscle
Anatomy of posterior interosseus nerve:
a. Passes through the supinator muscle
b. Innervates: All extrinsic wrist extensors except ECRL
i. Finger and thumb extensors - Most distal radial innervated muscle: extensor indicis proprius
ii. Extensor carpi ulnaris
iii. Abductor pollicis longus
iv. Sensory: Articular branches to wrist joint.

Ans. C : Radial nerve

ANATOMY
Ref.: BDC, 4th Ed., vol-I, page-90

43. Clavicle is generally said to have no medullary cavity, but this is not always true
Peculiarities of Clavicle:

 It has no medullary cavity

 It is the first bone to ossify in the fetus (5th-6th week)

 It is the only long bone having 2 primary centers of ossification (others have only 1)

 It is the only long bone that ossifies in membrane and not in cartilage

 It is the only long bone lying horizontally

 It is the most common fractured long bone in the body

 It is subcutaneous throughout

Ans. B: Clavicle
Ref.: BDC, 4th ed., Vol.-I, p-7; 5th ed., p-8

44. Vertebral level of the lower angle of scapula is D7

Other interesting facts related to scapula

 Winging of scapula: Injury to the thoracic nerve (of Bell). Prominence of medial border of scapula particularly on
pushing against the wall
Self Assessment & Review of FMGE / MCI Screening Examination
28

 Fracture of scapula: Rare, may be seen in violent trauma

 Sprengels shoulder: Scapula remains elevated, there is failure of descent

 Klippel-Feil deformity: Bilateral failure of descent of scapula. Webbing of neck and limitation of neck movements
due to failure of fusion of occipital bone and cervical spine defects is a feature

Ans. B: D7
Ref.: Maheshwari‖s Orthopaedics, 3rd ed., p-172t; Gray‖s Anatomy, 38th ed., p-1924

THORAX
45. • The spaces between the ribs contain three muscles of respiration: the external intercostal, the internal intercostal,
and the transversus thoracis muscle.
• The intercostal nerves and blood vessels run between the intermediate and deepest layers of muscles. They are
arranged in the following order from above downward: intercostal vein, intercostal artery, and intercostal nerve
(i.e., VAN).

Ans. A: VAN
Ref.: BDC 4th ed., vol.1, page-205, Clinical Anatomy-Snell, 8th ed., page-52
ANATOMY

46. Inhalation of foreign bodies into the lower respiratory tract is common, especially in children.

Parts of teeth may be inhaled while a patient is under anesthesia during a difficult dental extraction.

Because the right bronchus is the shorter (2.5 cm), wider and more direct continuation of the trachea, foreign bodies
tend to enter the right instead of the left bronchus. From there, they usually pass into the middle or lower lobe bronchi.

Ans. B: Lower lobe of right lung


Ref.: BDC 4th ed., vol.1, page-228, Clinical Anatomy-Snell, 8th ed., page-88

47. The visceral layer is closely applied to the heart and is often called the epicardium.

The slitlike space between the parietal and visceral layers is referred to as the pericardial cavity.

Normally, the cavity contains a small amount of tissue fluid (about 50 mL), the pericardial fluid, which acts as a
lubricant to facilitate movements of the heart.

Ans. A: 50 ml
Ref.: Clinical Anatomy-Snell, 8th ed., page-104

48. The arch of the aorta is a continuation of the ascending aorta.

It lies behind the manubrium sterni and arches upward, backward, and to the left in front of the trachea (its main
direction is backward).

It then passes downward to the left of the trachea and, at the level of the sternal angle, becomes continuous with the
descending aorta.
Anatomy 29

Branches

1. The brachiocephalic artery divides into the right subclavian and right common carotid arteries behind the right
sternoclavicular joint.

2. The left common carotid artery runs upward and to the left of the trachea and enters the neck behind the left
sternoclavicular joint.

3. The left subclavian artery runs upward along the left side of the trachea and the esophagus to enter the root of the
neck. It arches over the apex of the left lung.

Ans. C: Brachiocephalic artery


Ref.: BDC 4th ed., vol.1, page-261, Clinical Anatomy-Snell, 8th ed., page-125

49. • The trachea begins in the neck as a continuation of the larynx at the lower border of the cricoid cartilage at the level
of the sixth cervical vertebra.
• In the cadaver, trachea ends below at the carina by dividing into right and left principal (main) bronchi at the level
of the sternal angle (opposite the disc between the fourth and fifth thoracic vertebrae).
• In living subjects, in the erect posture, the bifurcation lies at the lower border of the sixth thoracic vertebra.
• In adults the trachea is about 10-15 cm long and 2 cm in diameter.
• The fibroelastic tube is kept patent by the presence of U-shaped bars (rings) of hyaline cartilage embedded in

ANATOMY
its wall.
• The posterior free ends of the cartilage are connected by smooth muscle, the trachealis muscle.

Ans. C: Opposite the disc between the T6-T7 vertebrae


Ref.: BDC 4th ed., vol.1, page-265

50. • The left coronary artery, which is usually larger than the right coronary artery, supplies the major part of the heart,
including the greater part of the left atrium, left ventricle, and ventricular septum.
• It arises from the left posterior aortic sinus of the ascending aorta.
• It then enters the atrioventricular groove and divides into an anterior interventricular branch and a circumflex
branch.
Branches

1. The anterior interventricular (descending) branch /left anterior descending (LAD) runs downward in the anterior
interventricular groove to the apex of the heart.

The anterior interventricular branch supplies the right and left ventricles with numerous branches that also supply
the anterior part of the ventricular septum. One of these ventricular branches (left diagonal artery) may arise
directly from the trunk of the left coronary artery.

2. The left circumflex artery (LCX) is the same size as the anterior interventricular artery. It winds around the left
margin of the heart in the atrioventricular groove. A left marginal artery is a large branch that supplies the left
margin of the left ventricle down to the apex. Anterior ventricular and posterior ventricular branches supply the
left ventricle. Atrial branches supply the left atrium.
Self Assessment & Review of FMGE / MCI Screening Examination
30

Posterior interventricular branch: It is typically a branch of the right coronary artery (80%, known as right
dominance). Alternately, the Posterior interventricular branch can be a branch of the left circumflex coronary artery
(20%, known as left dominance) which itself is a branch of the left coronary artery

Ans. D: Posterior interventricular branch


Ref.: BDC 4th ed., vol.1, page-250, Clinical Anatomy-Snell, 8th ed., page-113

51. • The arch of the aorta is a continuation of the ascending aorta.


• It lies behind the manubrium sterni and arches upward, backward, and to the left in front of the trachea (its main
direction is backward).
• It then passes downward to the left of the trachea and, at the level of the sternal angle, becomes continuous with the
descending aorta.

Branches

a. The brachiocephalic artery arises from the convex surface of the aortic arch.
It passes upward and to the right of the trachea and divides into the right subclavian and right common carotid
arteries behind the right sternoclavicular joint.
b. The left common carotid artery arises from the convex surface of the aortic arch on the left side of the
brachiocephalic artery.
• It runs upward and to the left of the trachea and enters the neck behind the left sternoclavicular joint.
ANATOMY

c. The left subclavian artery arises from the aortic arch behind the left common carotid artery.
• It runs upward along the left side of the trachea and the esophagus to enter the root of the neck.
• It arches over the apex of the left lung.

Ans. B: Right common carotid artery arises from the arch of aorta
Ref.: BDC 4th ed., vol.1, page-260, Clinical Anatomy-Snell, 8th ed., page-125

52. Aortic opening lies at the level of T12 vertebrae. It transmits:


i. Aorta
ii. Thoracic duct
iii. Azygous vein

Ans. D: Hemiazygous vein


Ref.: BDC 4th ed., vol.1, page-185

53. Ans. B: Left dominance


Ref.: BDC 4th ed., vol.1, page-250

54. The esophagus has four constrictions.


i. Where the pharynx joins the upper end - 15 cm from incisor teeth
ii. The second is at the where the aortic arch - 22.5 cm from incisor teeth
iii. The left bronchus cross its anterior surface - 27.5 cm from incisor teeth
iv. Where the esophagus passes through the diaphragm into the stomach - 37.5 cm from incisor teeth
These constrictions are of considerable clinical importance because they are sites where swallowed foreign bodies can
lodge or through which it may be difficult to pass an esophagoscope.
Anatomy 31

Because a slight delay in the passage of food or fluid occurs at these levels, strictures develop here after the drinking of
caustic fluids. Those constrictions are also the common sites of carcinoma of the esophagus.

Ans. D: At the point of crossing of thoracic duct


Ref.: BDC 4th ed., vol.1, page-268

55. Bochdalek hernia (involves an opening on the left side of the diaphragm) occur posteriorly and are due to a defect in
the posterior attachment of the diaphragm when there is a failure of pleuroperitoneal membrane closure in utero.

Retroperitoneal structures may prolapse through the defect, e.g. retroperitoneal fat, spleen or left kidney.

Bochdalek hernias occur more commonly on the posterior left side (85%, versus right side 15%).
Complications are usually due to pulmonary hypoplasia.
In adults, incidentally-discovered posterior diaphragmatic hernias are rare. Of these, right-sided hernias are more
common (68%), and more frequently in females.
The great majority are small, with only 27% containing abdominal organs such as bowel, spleen or liver.

Morgagni hernia – A Morgagni hernia involves an opening on the right side of the diaphragm. The liver and intestines
usually move up into the chest cavity.

Ans. B: Posterior and left


Ref.: BDC 4th ed., vol.2, page-312, Clinical Anatomy-Snell, 8th ed., page-62

ANATOMY
56. Coronary sinus receives blood mainly from the small, middle, great and oblique cardiac veins.
It also receives blood from the right marginal vein and the left posterior ventricular vein.
Most blood from the heart wall drains into the right atrium through the coronary sinus, which lies in the posterior part
of the atrioventricular.
It opens into the right atrium to the left of the inferior vena cava.
The anterior cardiac veins drain directly into the right atrium

Ans. A: Anterior cardiac vein


Ref.: BDC 4th ed., vol.1, page-251

57. Thoracic Part of the Sympathetic Trunk is continuous above with the cervical and below with the lumbar parts of the
sympathetic trunk.

It is the most laterally placed structure in the mediastinum and runs downward on the heads of the ribs.

It leaves the thorax on the side of the body of the 12th thoracic vertebra by passing behind the medial arcuate
ligament.

The sympathetic trunk has 12 (often only 11) segmentally arranged ganglia, each with white and gray ramus
communicans passing to the corresponding spinal nerve.

The first ganglion is often fused with the inferior cervical ganglion to form the stellate ganglion.
Branches
i. The postganglionic fibers are distributed through the branches of the spinal nerves to the blood vessels, sweat
glands, and erector pili muscles of the skin.
ii. The first five ganglia give postganglionic fibers to the heart, aorta, lungs, and esophagus.
Self Assessment & Review of FMGE / MCI Screening Examination
32

iii. The lower eight ganglia mainly give preganglionic fibers, which are grouped together to form the splanchnic
nerves and supply the abdominal viscera.

Ans. D: The first ganglion is often fused with the inferior cervical ganglion to form the stellate ganglion
Ref.: BDC 4th ed., vol.1, page-215, Clinical Anatomy-Snell, 8th ed., page-128

58. The esophagus is a muscular, collapsible tube about 10 in. (25 cm) long that joins the pharynx to the stomach.

The esophagus enters the abdomen through an opening in the right crus of the diaphragm.
After a course of about 0.5 in. (1.25 cm), it enters the stomach on its right side.

Ans. A: 25 cm
Ref.: BDC 4th ed., vol.1, page-267

59. The heart has three surfaces: sternocostal (anterior), diaphragmatic (inferior), and a base (posterior).
It also has an apex, which is directed downward, forward, and to the left.
The sternocostal surface is formed mainly by the right atrium and the right ventricle.
The right border is formed by the right atrium; the left border, by the left ventricle and part of the left auricle.
The diaphragmatic surface of the heart is formed mainly by the right and left ventricles. The inferior surface of the
right atrium, into which the inferior vena cava opens, also forms part of this surface.
The base of the heart, or the posterior surface, is formed mainly by the left atrium, into which open the four
ANATOMY

pulmonary veins.

Ans. B: Left atrium


Ref.: BDC 4th ed., vol.1, page-241, Clinical Anatomy-Snell, 8th ed., page-105

60. The lower border of the lung in midinspiration follows a curving line, which crosses the 6th rib in the
midclavicular line and the 8th rib in the midaxillary line, and reaches the 10th rib adjacent to the vertebral column
posteriorly.

Ans. B: 6th rib


Ref.: BDC 4th ed., vol.1, page-226, Clinical Anatomy-Snell, 8th ed., page-68

61. Trachea is kept patent by the presence of U-shaped bars (rings) of hyaline cartilage embedded in its wall. The
posterior free ends of the cartilage are connected by smooth muscle, the trachealis muscle.

Ans. C: C shaped
Ref.: BDC 4th ed., vol.1, page-266, Clinical Anatomy-Snell, 8th ed., page-87

62. The right coronary artery arises from the anterior aortic sinus of the ascending aorta and runs forward between the
pulmonary trunk and the right auricle.
The artery of the sinuatrial node (branch of right coronary artery) supplies the SA node and the right and left atria; in
40% of individuals it arises from the left coronary artery.

Ans. D: Right coronary artery


Ref.: BDC 4th ed., vol.1, page-249, Clinical Anatomy-Snell, 8th ed., page-113
Anatomy 33

63. Apart from the diaphragm and the intercostals, other less important muscles also contract on inspiration and assist in
elevating the ribs, namely, the levatores costarum muscles and the serratus posterior superior muscles.

Quiet expiration occurs passively by the elastic recoil of the pulmonary alveoli and thoracic wall.

In deep forced inspiration, a maximum increase in the capacity of the thoracic cavity occurs. Every muscle that can
raise the ribs is brought into action, including the scalenus anterior and medius and the sternocleidomastoid.
In respiratory distress the action of all the muscles already engaged becomes more violent, and the scapulae are fixed
by the trapezius, levator scapulae, and rhomboid muscles, enabling the serratus anterior and pectoralis minor to pull
up the ribs.

Forced expirartion is brought about by the muscles of the abdominal wall and the latissimus dorsi

Ans. D: None of the above


Ref.: BDC 4th ed., vol.1, page-203, Clinical Anatomy-Snell, 8th ed., page-102

64. The origin of the azygos vein is variable. It is often formed by the union of the right ascending lumbar vein and the
right subcostal vein.

It ascends through the aortic opening in the diaphragm on the right side of the aorta to the level of the fifth thoracic
vertebra.

ANATOMY
Here it arches forward above the root of the right lung to empty into the posterior surface of the superior vena cava.

The azygos vein has numerous tributaries, including the fifth to eleventh right posterior intercostal veins, the right
superior intercostal vein, the hemiazygos and the accessory hemiazygos veins, and numerous esophagral, mediastinal
and pericardial veins.

Ans. B: Superior vena cava


Ref.: BDC 4th ed., vol.1, page-213, Clinical Anatomy-Snell, 8th ed., page-123

65. In 85% of patients the right coronary artery (RCA) is said to be “dominant” because it supplies circulation to the
inferior portion of the interventricular septum via the right posterior descending coronary artery/posterior
interventricular artery.
In these cases the RCA travels to the cross-section of the AV groove and the posterior interventricular (IV groove).
Here, it gives rise to the right posterior descending coronary artery (PDA) branch which travels in the posterior IV
groove and gives off several septal perforator branches (SP). The SP supply blood to the lower portion of the IV
septum.
Generally, the dominant RCA also gives rise to the AV nodal branch which supplies blood to the AV node
The dominant RCA also provides the right postero-lateral (PLA) branch to the lower postero-lateral portion of the left
ventricle.
The sinus or sino-atrial (SA) node branch originates in the proximal portion of the RCA in 60% of cases and as a left
atrial branch of the Cx in the remaining 40% of cases. This is unrelated to whether the artery is “dominant” or not.

Ans. B: Supplying circulation to the inferior portion of the interventricular septum


Ref.: BDC 4th ed., vol.1, page-250
Self Assessment & Review of FMGE / MCI Screening Examination
34

66. If the superior or inferior vena cava is obstructed, the venous blood causes distention of the veins running from the
anterior chest wall to the thigh.

The lateral thoracic vein anastomoses with the superficial epigastric vein, a tributary of the great saphenous vein of the
leg. In these circumstances, a tortuous varicose vein may extend from the axilla to the lower abdomen
The most common cause of superior vena cava syndrome is cancer.
i. Primary or metastatic cancer in the upper lobe of the right lung can compress the superior vena cava.
ii. Lymphoma or other tumors located in the mediastinum can also cause compression of the superior vena cava.
iii. Less often, the superior vena cava can become blocked with a blood clot from within.
iv. invasive medical procedures (Blood clot (thrombus) formation that causes superior vena cava syndrome is a
complication of pacemaker wires, dialysis, and other intravenous catheters that are threaded into the superior vena cava)
v. infection (syphilis and tuberculosis) is another cause of superior vena cava syndrome. Sarcoidosis (a disease that
results in masses of inflamed tissue) may also cause this syndrome.

Ans. C: Lung cancer


Ref.: BDC 4th ed., vol.1, page-258, Clinical Anatomy-Snell, 8th ed., page-162

67. The arch of the aorta (Transverse Aorta) begins at the level of the upper border of the second sternocostal articulation
of the right side, and runs at first upward, backward, and to the left in front of the trachea; it is then directed backward
ANATOMY

on the left side of the trachea and finally passes downward on the left side of the body of the fourth thoracic vertebra,
at the lower border of which it becomes continuous with the descending aorta.

Ans. C: T4
Ref.: BDC‖s Anatomy, Vol-I, 4th ed.,p-260

68. Right border of the mediastinal shadow (chiefly produced due to the heart and the vessels entering and leaving it) is formed from
above downwards by the right brachiocephalic vein, superior vena cava, right atrium and the inferior vena cava

The Base of the Heart

 The base is located posteriorly and is formed mainly by the left atrium.
 It lies opposite T5 to T8 (supine position) and T6 to T9 vertebrae (erect position) and faces superiorly, posteriorly
and towards the right shoulder.
 The base or posterior aspect of the heart is quadrilateral in shape and it is from its most superior part from which
the ascending aorta and pulmonary trunk emerge, and into which the superior vena cava enters.
 The base is separated from the diaphragmatic surface of the heart by the posterior part of the coronary groove
(L. sulcus).
 The heart does not rest on its base. The term refers to the somewhat conical shape of the heart with the base being
opposite the apex.

The Apex of the Heart


 This blunt apex is formed from by the left ventricle, which points inferolaterally.
 The apex is located posterior to the left 5th intercostal space in adults, 7 to 9 cm from the median plane, and just
left of the midclavicular line.
Anatomy 35

 The apex beat is an impulse imparted by the hear; it is its point of maximal pulsation or the lowest, most lateral
point at which pulsation can be felt.
The Sternocostal (Anterior) Surface of the Heart
 This surface of the heart is mainly formed by the right ventricle and is visible in PA radiographs of the thorax.
The Diaphragmatic (Inferior) Surface of the Heart
 The surface of the heart is usually horizontal or slightly concave.
 It is formed by both ventricles, mainly the left one.
 The posterior interventricular sulcus divides this surface into a right 1/3 and a left 2/3.
The Pulmonary (Left) Surface of the Heart
 This surface of the heart is mainly formed by the left ventricle and occupies the cardiac notch of the left lung.
The Borders of the Heart

 The heart has four borders: right, inferior, left, and superior.
 These borders are actually the borders of its sternocostal surface.
 The right border is formed by the right atrium.It is slightly convex and is almost in line with the superior and
inferior vena cavae.
 The inferior border is sharp and thin, and nearly horizontal. It is formed mainly by the right ventricle and only
slightly by the left ventricle.

ANATOMY
 The left border is formed mainly by the left ventricle and only slightly by the left auricle.
 The superior border is where the great vessels enter and leave the heart. It is formed by the right and left auricles
and the superior conical portion of the right ventricle, the conus arteriosus (infundibulum), between them.
 The pulmonary trunk arises from the conus arteriosus.
Radiological Anatomy of the Heart
 In PA radiographs of the thorax, the right border of the cardiovascular silhouette is formed by (superior to
inferior): (1) the superior vena cava; (2) the right atrium; (3) the inferior vena cava.
 The left border of the cardiovascular silhouette is formed by: (1) the arch of the aorta, which produces a
characteristic aortic knob; (2) the pulmonary trunk; (3) the left auricle; (4) the left ventricle.

Ans. C: Superior vena cava


Ref.: BDC 4th ed., vol.1, page-258, Clinical Anatomy-Snell, 8th ed., page-162

69. Venacaval opening lies in the central tendon of the diaphragm at the level of T8 and it transmits inferior vena cava and branches of
right phrenic nerve

Diaphragm

 T8 Level: Caval hiatus (through central tendon of the diaphragm) transmitting the inferior vena cava, branches of
right phrenic nerve
 T9 Level: Foramen of Morgagni also called sternocostal hiatus two on each side of the xiphoid process.
Transmitting the superior epigastric vessels.
 T10 Level: Esophageal hiatus (through muscular part) transmitting the esophagus, gastric (vagus) nerve and
esophageal branches of the left gastric artery and accompanying veins
 T12 Level: Aortic hiatus (osseoaponeurotic) transmitting the aorta, the azygous vein, and the thoracic duct.
Self Assessment & Review of FMGE / MCI Screening Examination
36

 A commonly used mnemonic to remember the level of the diaphragmatic apertures is this: Mnemonic
 Aortic hiatus = 12 letters = T12
 Oesophagus = 10 letters = T10
 Vena cava = 8 letters = T8

Embryology:

 The central tendinous portions are derived from the pleuroperitoneal folds and the septum transversum.
 While the crura are derived from the dorsal esophageal mesentry, the peripheral muscular portions of the
diaphragm are derived from the body wall.

Ans. B: Right Phrenic nerve


Ref.: BDC, 4th ed., Vol.-II, p-185; 5th ed., p-188,189

70. The azygos vein ends by joining the posterior aspect of the superior vena cava
The Azygos Vein
 The azygos vein connects the superior and inferior venae cavae, either directly by joining the IVC or indirectly by
the hemiazygos and accessory hemiazygos veins.
 The azygos vein drains blood from the posterior walls of the thorax and abdomen.
 It ascends in the posterior mediastinum, passing close to the right sides of the bodies of the inferior eight
thoracic vertebrae (T4-T12).
ANATOMY

 It is covered anteriorly by the oesophagus as it passes posterior to the root of the right lung.
 It then arches over the superior aspect of this root to join the SVC.
 In addition to the posterior intercostal veins, the azygos vein communicates with the vertebral venous plexuses.
 This vein also receives the mediastinal, oesophageal, and bronchial veins.

Ans. C: Superior vena cava


Ref.: BDC/I, 5th ed., p-218

INFERIOR EXTREMITY
71. The talocalcaneonavicular and the calcaneocuboid joints are together referred to as the midtarsal or transverse tarsal
joints.
The important movements of inversion and eversion of the foot take place at the subtalar and transverse tarsal joints
Inversion is the movement of the foot so that the sole faces medially.
Eversion is the opposite movement of the foot so that the sole faces in the lateral direction.
Inversion is performed by the tibialis anterior, the extensor hallucis longus, and the medial tendons of extensor
digitorum longus; the tibialis posterior also assists.
Eversion is performed by the peroneus longus, peroneus brevis, and peroneus tertius; the lateral tendons of the
extensor digitorum longus also assist.
Ankle joint’s active movements are dorsiflexion and plantar flexion
Inferior Tibiofibular joint permits slight movements so that the lateral malleolus can rotate laterally during
dorsiflexion of the ankle

Ans. A: Subtalor joints


Ref.: BDC 4th ed., vol.2, page-154,153,152, Clinical Anatomy-Snell, 8th ed., page-638
Anatomy 37

72. In patients with occlusive coronary disease caused by atherosclerosis, the diseased arterial segment can be bypassed
by inserting a graft consisting of a portion of the great saphenous vein.
The venous segment is reversed so that its valves do not obstruct the arterial flow. Following removal of the great
saphenous vein at the donor site, the superficial venous blood ascends the lower limb by passing through perforating
veins and entering the deep veins.

The great saphenous vein can also be used to bypass obstructions of the brachial or femoral arteries.

Ans. A: Great saphenous vein


Ref.: Clinical Anatomy-Snell, 8th ed., page-572

73. Other attachments on the ischial tuberosity are:


1. Long head of biceps femoris
2. Sacrotuberous ligament
3. Ischiofemoral ligament
Adductor longus arises from the body of pubis

Ans. A: Adductor longus


Ref.: BDC 4th ed., vol.2, page-13,11

ANATOMY
74. Deep Fascia of the Thigh‖s (Fascia Lata) upper end is attached to the pelvis and the inguinal ligament.

On its lateral aspect, it is thickened to form the iliotibial tract, which is attached above to the iliac tubercle and below
to the lateral condyle of the tibia. The iliotibial tract receives the insertion of the tensor fasciae latae and the greater
part of the gluteus maximus muscle.

In the gluteal region, the deep fascia forms sheaths, which enclose the tensor fasciae latae and the gluteus maximus
muscles.
The saphenous opening is a gap in the deep fascia in the front of the thigh just below the inguinal ligament. It
transmits the great saphenous vein, some small branches of the femoral artery, and lymph vessels.
The saphenous opening is filled with loose connective tissue called the cribriform fascia.

Ans. C: Medially the fascia is thickened to form iliotibial tract


Ref.: BDC 4th ed., vol.2, page-49, Clinical Anatomy-Snell, 8th ed., page-573

75. Abdominal herniae are of the following common types:


i. Inguinal (indirect or direct)
ii. Femoral
iii. Umbilical (congenital or acquired)
iv. Epigastric
v. Separation of the recti abdominis
vi. Incisional
vii. Hernia of the linea semilunaris (Spigelian hernia)
viii. Lumbar (Petit‖s triangle hernia)
Self Assessment & Review of FMGE / MCI Screening Examination
38

Hernial sac in indirect inguinal hernia may extend through the superficial inguinal ring above and medial to the pubic
tubercle whereas in femoral hernias the sac is located below and lateral to the pubic tubercle
The indirect inguinal hernia can be summarized as follows:
 It is the remains of the processus vaginalis and therefore is congenital in origin.
 It is more common than a direct inguinal hernia.
 It is much more common in males than females.
 It is more common on the right side.
 It is most common in children and young adults.
The hernial sac enters the inguinal canal through the deep inguinal ring and lateral to the inferior epigastric vessels.
The neck of the sac is narrow.
The hernial sac may extend down into the scrotum or labium majus.
A direct inguinal hernia can be summarized as follows:
It is common in old men with weak abdominal muscles and is rare in women.
The hernial sac bulges forward through the posterior wall of the inguinal canal medial to the inferior epigastric
vessels.
The neck of the hernial sac is wide.

Ans. B: Below and lateral


Ref.: Clinical Anatomy-Snell, 8th ed., page-185
ANATOMY

76. Pulsations of the femoral artery can be felt at the midinguinal point against the head of the femur
Femoral artery
 It begins immediately behind the inguinal ligament, midway between the anterior superior spine of the ilium and
the symphysis pubis, and passes down the front and medial side of the thigh (hence palpated in this region).
 It ends at the junction of the middle with the lower third of the thigh, where it passes through an opening in the
Adductor magnus to become the popliteal artery.
 The vessel, at the upper part of the thigh, lies in front of the hip-joint; in the lower part of its course it lies to the
medial side of the body of the femur, and between these two parts, where it crosses the angle between the head
and body, the vessel is some distance from the bone.
 The first 4 cm. of the vessel is enclosed, together with the femoral vein, in a fibrous sheath—the femoral sheath.
 In the upper third of the thigh the femoral artery is contained in the femoral triangle (Scarpa’s triangle), and in the
middle third of the thigh, in the adductor canal (Hunter’s canal).

Ans. D: Mid-inguinal point


Ref.: BDC, 3rd ed., Vol.-II, p-48; 5th ed, p-61

77. Prepatellar’s bursitis is called “housemaids knee” or miner’s knee

Ans. C: Prepatellar bursa


Ref.: BDC, 4th ed., Vol.-II, p-48; 5th ed, p-63

78. Prepatellar’s bursitis is called “housemaids knee” or miner’s knee


Prepatellar bursitis
 It is a common cause of swelling and pain above the patella
 It is due to inflammation of the prepatellar bursa.
Anatomy 39

 This structure is a superficial bursa with a thin synovial lining located between the skin and the patella.
 The bursa develops within the first years of life as a result of mechanical pressure and friction, and it serves the
purpose of reducing friction on underlying structures and allowing maximal range of motion in the knee.
 Aseptic prepatellar bursitis is commonly caused by repetitive work in a kneeling position, hence the name
"housemaid's knee".
Infrapatellar bursitis/ clergyman's knee

 It is the inflammation of the infrapatellar bursa, which is located just below the patella.
It is often called "clergyman's knee" due to its historical frequency amongst clergyman, who injured the bursa by
kneeling on hard surfaces during prayer.

Ans. B: Prepatellar bursa


Ref.: BDC, 4th ed., Vol-II, p-48

ABDOMEN
79. The branches of these divisions supply the pelvic viscera, the perineum, the pelvic walls, and the buttocks.

Branches of the Anterior Division:

1. Umbilical artery: From the proximal patent part of the umbilical artery arises the superior vesical artery, which
supplies the upper portion of the bladder.

ANATOMY
2. Uterine artery: It ends by following the uterine tube laterally, where it anastomoses with the ovarian artery.

3. Vaginal artery: This artery usually takes the place of the inferior vesical artery present in the male. It supplies the
vagina and the base of the bladder.

4. Obturator artery: This artery leaves the pelvis through the obturator canal.

5. Middle rectal artery: Commonly, this artery arises with the inferior vesical artery.

6. Internal pudendal artery: This artery leaves the pelvis through the greater sciatic foramen and enters the gluteal
region below the piriformis muscle.
7. Inferior gluteal artery: This artery leaves the pelvis through the greater sciatic foramen below the piriformis muscle.

8. Inferior vesical artery: This artery supplies the base of the bladder and the prostate and seminal vesicles in the
male; it also gives off the artery to the vas deferens.

Branches of the Posterior Division

1. Iliolumbar artery

2. Lateral sacral arteries

3. Superior gluteal artery: This artery leaves the pelvis through the greater sciatic foramen above the piriformis
muscle. It supplies the gluteal region.
Ovarian artery arises from the abdominal aorta below the renal artery.

Ans. A: Ovarian artery


Ref.: BDC 4th ed., vol.2, page-387, Clinical Anatomy-Snell, 8th ed., page-328
Self Assessment & Review of FMGE / MCI Screening Examination
40

80. • The uterus is supported mainly by the tone of the levator ani muscles and the condensations of pelvic fascia, which
form three important ligaments.

The Levator Ani Muscles and the Perineal Body

• They form a broad muscular sheet. They effectively support the pelvic viscera. The medial edges of the anterior
parts of the levator ani muscles are attached to the cervix of the uterus by the pelvic fascia.

• Some of the fibers of levator ani are inserted into a fibromuscular structure called the perineal body. This structure
is important in maintaining the integrity of the pelvic floor; if the perineal body is damaged during childbirth,
prolapse of the pelvic viscera may occur.

Transverse Cervical (Cardinal) Ligaments

• Transverse cervical ligaments are fibromuscular condensations of pelvic fascia that pass to the cervix and the upper
end of the vagina from the lateral walls of the pelvis.

Pubocervical Ligaments

• The pubocervical ligaments consist of two firm bands of connective tissue that pass to the cervix from the posterior
surface of the pubis. They are positioned on either side of the neck of the bladder, to which they give some support
(pubovesical ligaments).
ANATOMY

Sacrocervical Ligaments

• The sacrocervical ligaments consist of two firm fibromuscular bands of pelvic fascia that pass to the cervix and the
upper end of the vagina from the lower end of the sacrum. They form two ridges, one on either side of the
rectouterine pouch (pouch of Douglas).

• The broad ligaments and the round ligaments of the uterus are lax structures, and the uterus can be pulled up or
pushed down for a considerable distance before they become taut. Clinically, they are considered to play a minor
role in supporting the uterus.

• The round ligament of the uterus, which represents the remains of the lower half of the gubernaculum, helps keep
the uterus anteverted (tilted forward) and anteflexed (bent forward) but is considerably stretched during
pregnancy.

Ans. C: Broad ligament


Ref.: BDC 4th ed., vol.2, page-361, Clinical Anatomy-Snell, 8th ed., page-368

81. Superficial Inguinal Lymph Nodes


1. The horizontal group

2. The medial members of the group receive superficial lymph vessels from the anterior abdominal wall below the
level of the umbilicus and from the perineum. The lymph vessels from the urethra, the external genitalia of both
sexes (but not the testes), and the lower half of the anal canal are drained by this route.

3. The lateral members of the group receive superficial lymph vessels from the back below the level of the iliac crests.

4. The vertical group lies along the terminal part of the great saphenous vein and receives most of the superficial
lymph vessels of the lower limb.
Anatomy 41

The efferent lymph vessels from the superficial inguinal nodes pass through the saphenous opening in the deep fascia
and join the deep inguinal nodes.

Deep Inguinal Lymph Nodes

The deep nodes are located beneath the deep fascia and lie along the medial side of the femoral vein; the efferent
vessels from these nodes enter the abdomen by passing through the femoral canal to lymph nodes along the external
iliac artery

Lymphatic Drainage of the Penis

From most of the penis, lymph drains into the superficial inguinal lymph nodes.

Vessels from the glans penis drain into the deep inguinal lymph nodes.

Ans. C: Glans penis


Ref.: BDC 4th ed., vol.2, page-133, Clinical Anatomy-Snell, 8th ed., page-573

82. Ans. A: Renal fascia


Ref.: BDC 4th ed., vol.2, page-297, Dorlands med dictionary 28th ed. p-609

83. The inguinal canal is an oblique passage through the lower part of the anterior abdominal wall.

ANATOMY
The canal is about 1.5 in. (4 cm) long in the adult and extends from the deep inguinal ring, a hole in the fascia
transversalis, downward and medially to the superficial inguinal ring, a hole in the aponeurosis of the external
oblique muscle.

In the males, it allows structures to pass to and from the testis to the abdomen.

In females it allows the round ligament of the uterus to pass from the uterus to the labium majus.

Ans. B: External oblique aponeurosis


Ref.: BDC 4th ed., vol.2, page-208, Clinical Anatomy-Snell, 8th ed., page-164

84. • Rectouterine pouch (pouch of Douglas) is the most dependent part of the entire peritoneal cavity (when the
patient is in the standing position), hence it frequently becomes the site for the accumulation of blood (from a
ruptured ectopic pregnancy) or pus (from a ruptured pelvic appendicitis or in gonococcal peritonitis).

• Because the pouch lies directly behind the posterior fornix of the vagina, it is commonly violated by misguided
nonsterile instruments, which pierce the wall of the posterior fornix in a failed attempt at an illegal abortion.

• A needle may be passed into the pouch through the posterior fornix in the procedure known as culdocentesis.

• Surgically, the pouch may be entered in posterior colpotomy. The interior of the female pelvic peritoneal cavity
may be viewed for evidence of disease through an endoscope.

Ans. D: Rectum and Uterus


Ref.: BDC 4th ed., vol.2, page-234, Clinical Anatomy-Snell, 8th ed., page-376
Self Assessment & Review of FMGE / MCI Screening Examination
42

85. • An extensive venous plexus, the pampiniform plexus, leaves the posterior border of the testis.

• As the plexus ascends, it becomes reduced in size so that at about the level of the deep inguinal ring, a single
testicular vein is formed.

• This runs up on the posterior abdominal wall and drains into the left renal vein on the left side and into the
inferior vena cava on the right side.

Ans. B: Left renal vein


Ref.: BDC 4th ed., vol.2, page-218, Clinical Anatomy-Snell, 8th ed., page-165

86. The superior mesenteric artery (SMA) arises from the anterior surface of abdominal aorta, just inferior to the origin of
the celiac trunk, and supplies the intestine from the lower part of the duodenum to the left colic flexure and the
pancreas.
Branches of SMA
i. Intestinal arteries (arcades—vasa recta—straight arteries) gives branches to ileum, branches to jejunum-(terminal
branch of the SMA) supplies last part of ileum, cecum, and appendix
ii. Ileocolic artery
a. appendicular artery
iii. Right Colic artery supplies ascending colon.
ANATOMY

iv. Middle Colic artery supplies the transverse colon.


v. Inferior pancreaticoduodenal artery supplies head of the pancreas and to the descending and inferior parts of the
duodenum

Ans. D: Descending colon


Ref.: BDC 4th ed., vol.2, page-264

87. The uterus is chiefly supplied by the two uterine arteries and partly by ovarian arteries

Ans. C: Both
Ref.: BDC 4th ed., vol.2, page-360

88. • Branches of the sacral plexus, the pudendal nerve, and nerve to the obturator internus leave the pelvis through the
lower part of the greater sciatic foramen, below the piriformis.
• They cross the ischial spine with the internal pudendal artery and immediately re-enter the pelvis through the
lesser sciatic foramen; they then lie in the ischiorectal fossa.
• The pudendal nerve supplies structures in the perineum.
• The nerve to the obturator internus supplies the obturator internus muscle on its pelvic surface.

Ans. A: Ischial spine


Ref.: BDC 4th ed., vol.2, page-335, Clinical Anatomy-Snell, 8th ed., page-566

89. The kidneys have the following coverings:


i. Fibrous capsule: This surrounds the kidney and is closely applied to its outer surface.
ii. Perirenal fat: This covers the fibrous capsule.
Anatomy 43

iii. Renal fascia: This is a condensation of connective tissue that lies outside the perirenal fat and encloses the kidneys
and suprarenal glands; it is continuous laterally with the fascia transversalis.
iv. Pararenal fat: This lies external to the renal fascia and is often in large quantity.
It forms part of the retroperitoneal fat.
The perirenal fat, renal fascia, and pararenal fat support the kidneys and hold them in position on the posterior
abdominal wall.

Ans. D: Fibrous capsule


Ref.: BDC 4th ed., vol.2, page-297, Clinical Anatomy-Snell, 8th ed., page-262

90. Ans. D: Internal iliac artery


Ref.: BDC 4th ed., vol.2, page-360, Clinical Anatomy-Snell, 8th ed., page-328

91. The superficial inguinal ring is a triangular aperture in the aponeurosis of the external oblique muscle and is situated
above and medial to the pubic tubercle.
In the female, the superficial inguinal ring is smaller and difficult to palpate; it transmits the round ligament of the
uterus.

Ans. B: Round ligament of the uterus


Ref.: BDC 4th ed., vol.2, page-208, Clinical Anatomy-Snell, 8th ed., page-191

ANATOMY
92. The kidneys are reddish brown and lie behind the peritoneum high up on the posterior abdominal wall on either side
of the vertebral column; they are largely under cover of the costal margin
The right kidney lies slightly lower than the left kidney because of the large size of the right lobe of the liver.
On the medial concave border of each kidney is a vertical slit that is bounded by thick lips of renal substance and is
called the hilum.
The hilum transmits, from the front backward, the renal vein, two branches of the renal artery, the ureter, and the
third branch of the renal artery (VAUA). Lymph vessels and sympathetic fibers also pass through the hilum.

Ans. C: Left kidney is situated lower than the right


Ref.: BDC 4th ed., vol.2, page-296, Clinical Anatomy-Snell, 8th ed., page-260

93. The two uterine tubes are each about 4 in. (10 cm) long and lie in the upper border of the broad ligament.
Each connects the peritoneal cavity in the region of the ovary with the cavity of the uterus.
The uterine tube is divided into four parts:
The infundibulum is the funnel-shaped lateral end that projects beyond the broad ligament and overlies the ovary.
The tubal ostium is the point where the tubal canal meets the peritoneal cavity.
The ampulla is the widest part of the tube.
The isthmus is the narrowest part of the tube and lies just lateral to the uterus.
The intramural part is the segment that pierces the uterine wall.

Function

The uterine tube receives the ovum from the ovary and provides a site where fertilization of the ovum can take place
(usually in the ampulla).
Self Assessment & Review of FMGE / MCI Screening Examination
44

The inner mucous membrane of the uterine tube is lined by the ciliated columnar epithelium mixed with the
nonciliated secretory cells or peg cells
The Müllerian ducts develops in females into the fallopian tubes, uterus and vagina, while the Wolffian ducts
develops in males into the epididymis and vas deferens

Ans. A: Lined by cuboidal epithelium


Ref.: BDC 4th ed., vol.2, page-357, Clinical Anatomy-Snell, 8th ed., page-363

94. The left gastric artery arises from the celiac artery. It supplies the lower third of the esophagus and the upper right
part of the stomach.
The right gastric artery arises from the hepatic artery at the upper border of the pylorus and runs to the left along the
lesser curvature. It supplies the lower right part of the stomach.
The short gastric arteries arise from the splenic artery at the hilum of the spleen and pass forward in the
gastrosplenic omentum (ligament) to supply the fundus.
The left gastroepiploic artery arises from the splenic artery at the hilum of the spleen and passes forward in the
gastrosplenic omentum (ligament) to supply the stomach along the upper part of the greater curvature.
The right gastroepiploic artery arises from the gastroduodenal branch of the hepatic artery. It passes to the left and
supplies the stomach along the lower part of the greater curvature.

Ans. B: Splenic artery


ANATOMY

Ref.: BDC 4th ed., vol.2, page-264, Clinical Anatomy-Snell, 8th ed., page-220

95. The ovarian artery arises from the abdominal part of the aorta at the level of the first lumbar vertebra.
The artery is long and slender and passes downward and laterally behind the peritoneum. It crosses the external iliac
artery at the pelvic inlet and enters the suspensory ligament of the ovary.
It then passes into the broad ligament and enters the ovary by way of the mesovarium.

Ans. C: Abdominal part of the aorta


Ref.: BDC 4th ed., vol.2, page-315, Clinical Anatomy-Snell, 8th ed., page-328

96. The vermiform appendix is located in the right lower quadrant of abdomen.
It is a narrow, worm shaped tube, arising from the posteromedial caecal wall, 2 cms or less below the end of the ileum.
Its opening is occasionally guarded by a semicircular fold of mucous membrane known as the valve of Gerlach.
The appendix is usually located at the junction of the taeniae, found on the surface of the caecum.
Its length varies from 2-20 cms, with an average length of 9 cms.
The attachment of the base of the appendix to the caecum remains constant, whereas the tip can be found in a
retrocaecal (65%)-commonest, pelvic (30%)-second most common, subcaecal, preileal, post-ileal or promontoric
positions.
The mesoappendix has a free border which carries the blood supply to the organ, by the appendicular artery, a branch
from the ileocolic.
The appendix develops from the midgut loop together with the caecum, ascending colon and the proximal two thirds
of the transverse colon.
Appendicitis is the most common cause of acute abdomen in young people.

Ans. B: Retrocaecal
Ref.: BDC 4th ed., vol.2, page-256, Clinical Anatomy-Snell, 8th ed., page-232
Anatomy 45

97 The portal vein drains blood from the abdominal part of the gastrointestinal tract from the lower third of the
esophagus to halfway down the anal canal; it also drains blood from the spleen, pancreas, and gallbladder.
The tributaries of the portal vein are the splenic vein, superior mesenteric vein, left gastric vein, right gastric vein,
superior pancreaticoduodenal, paraumbilical and cystic veins.
The portal vein enters the liver and breaks up into sinusoids, from which blood passes into the hepatic veins that join
the inferior vena cava. The portal vein is about 2 in. (5 cm) long and is formed behind the neck of the pancreas by the
union of the superior mesenteric and splenic veins.
Renal veins join the inferior vena cava just below the transpyloric plane

Ans. A: Renal vein


Ref.: BDC 4th ed., vol.2, page-270,316, Clinical Anatomy-Snell, 8th ed., page-245

98. Relations of stomach


Anteriorly: The anterior abdominal wall, the left costal margin, the left pleura and lung, the diaphragm, and the left
lobe of the liver
Posteriorly: The lesser sac, the diaphragm, the spleen, the left suprarenal gland, the upper part of the left kidney,
the splenic artery, the pancreas, the transverse mesocolon, and the transverse colon.
Hepatic flexure of colon is related to the colic impression on the inferior surface of the right lobe of the liver

ANATOMY
Ans. A: Hepatic flexure of colon
Ref.: BDC 4th ed., vol.2, page-240,258, Clinical Anatomy-Snell, 8th ed., page-220

99. Lymph Drainage of ovary


The lymph vessels of the ovary follow the ovarian artery and drain into the pre aortic and para-aortic nodes at the
level of the first lumbar vertebra.
Arterial Supply
The ovarian artery arises from the abdominal aorta at the level of the first lumbar vertebra.
Venous drainage.
The ovarian vein drains into the inferior vena cava on the right side and into the left renal vein on the left side.

Nerve Supply

The nerve supply to the ovary is derived from the aortic plexus and accompanies the ovarian artery.

Ans. A: Preaortic and para-aortic lymph nodes


Ref.: BDC 4th ed., vol.2, page-355,133, Clinical Anatomy-Snell, 8th ed., page-361

100. Blood supply of testes


The testicular artery is a branch of the abdominal aorta given off at the level of L2.
The testicular veins emerge from the testis and the epididymis as a venous network, the pampiniform plexus.
This becomes reduced to a single vein as it ascends through the inguinal canal. The right testicular vein drains into the
inferior vena cava, and the left vein joins the left renal vein.

Ans. D: Testicular artery


Ref.: BDC 4th ed., vol.2, page-218, Clinical Anatomy-Snell, 8th ed., page-169
Self Assessment & Review of FMGE / MCI Screening Examination
46

101. Normally the long axis of the uterus is bent forward on the long axis of the vagina. This position is referred to as
anteversion of the uterus.
The long axis of the body of the uterus is bent forward at the level of the internal os with the long axis of the cervix.
This position is termed anteflexion of the uterus.
Thus, in the erect position and with the bladder empty, the uterus lies in an almost horizontal plane.
If the fundus and body of the uterus are bent backward on the vagina so that they lie in the rectouterine pouch (pouch
of Douglas), the uterus is said to be retroverted.
If the body of the uterus is, in addition, bent backward on the cervix, it is said to be retroflexed.

Ans. A: Normally the uterus is retroverted


Ref.: BDC 4th ed., vol.2, page-358, Clinical Anatomy-Snell, 8th ed., page-366

102. The spleen is the largest single mass of lymphoid tissue in the body.
It lies just beneath the left half of the diaphragm close to the 9th, 10th, and 11th ribs.
The long axis lies along the shaft of the 10th rib, and its lower pole extends forward only as far as the midaxillary line.
The spleen is surrounded by peritoneum which passes from it at the hilum as the gastrosplenic omentum (ligament)
to the greater curvature of the stomach (carrying the short gastric and left gastroepiploic vessels).
The peritoneum also passes to the left kidney as the splenicorenal/lineorenal ligament (carrying the splenic vessels
ANATOMY

and the tail of the pancreas).


Phrenicocolic ligament is not attached to the spleen but supports its anterior end
Ligamentum teres is related with the liver and represents the obliterated left umbilical vein

Ans. D: Ligamentum teres


Ref.: BDC 4th ed., vol.2, page-281, Clinical Anatomy-Snell, 8th ed., page-259

103. Each ureter measures about 10 in. (25 cm) long and resembles the esophagus (also 10 in. long) in having three
constrictions along its course: where the renal pelvis joins the ureter, where it is kinked as it crosses the pelvic brim,
and where it pierces the bladder wall.

The ureter emerges from the hilum of the kidney and runs vertically downward behind the parietal peritoneum
(adherent to it) on the psoas muscle, which separates it from the tips of the transverse processes of the lumbar
vertebrae.

It enters the pelvis by crossing the bifurcation of the common iliac artery in front of the sacroiliac joint .

The ureter then runs down the lateral wall of the pelvis to the region of the ischial spine and turns forward to enter the
lateral angle of the bladder.
Relations, Right Ureter
Anteriorly: The duodenum, the terminal part of the ileum, the right colic and ileocolic vessels, the right testicular or
ovarian vessels, and the root of the mesentery of the small intestine.
Posteriorly: The right psoas muscle, which separates it from the lumbar transverse processes, and the bifurcation of the
right common iliac artery.
Anatomy 47

Relations, Left Ureter


Anteriorly: The sigmoid colon and sigmoid mesocolon, the left colic vessels, and the left testicular or ovarian vessels.
Posteriorly: The left psoas muscle, which separates it from the lumbar transverse processes, and the bifurcation of the
left common iliac artery.
The inferior mesenteric vein lies along the medial side of the left ureter
Ureters are lined by transitional epithelium.

Ans. D: It is lined by cuboidal epithelium


Ref.: BDC 4th ed., vol.2, page-301,304, Clinical Anatomy-Snell, 8th ed., page-266

104. Transpyloric Plane passes through the tips of the ninth costal cartilages on the two sides that is, the point where the
lateral margin of the rectus abdominis (linea semilunaris) crosses the costal margin.
The transpyloric plane is clinically notable because it passes through several important abdominal structures.
These include:
a. Lumbar vertebra 1 and hence passes just before the end of the spinal cord in adults
b. The fundus of the gallbladder
c. The neck of the pancreas
d. The pancreatic body

ANATOMY
e. The origins of the superior mesenteric artery from the aorta and portal vein
f. The left and right colic flexure
g. The left hilum of the kidney
h. The right hilum of the kidney
i. The root of the transverse mesocolon
j. Duodenojejunal flexure
k. The 2nd part of the duodenum
l. The upper part of conus medullaris
m. The spleen

Ans. B: Fundus of stomach


Ref.: BDC 4th ed., vol.2, page-194,221, Clinical Anatomy-Snell, 8th ed., page-192

105. Relations of spleen:


Anteriorly: The stomach, tail of the pancreas, and left colic flexure. The left kidney lies along its medial border.
Posteriorly: The diaphragm; left pleura (left costodiaphragmatic recess); left lung; and 9th, 10th, and 11th rib.

Ans. D: The stomach


Ref.: BDC 4th ed., vol.2, page-281,282, Clinical Anatomy-Snell, 8th ed., page-259

106. Veins which do not have valves are:


1. IVC
2. SVC
Self Assessment & Review of FMGE / MCI Screening Examination
48

3. Hepatic, ovarian, uterine, renal, emissary, cerebral, pulmonary and umbilical veins
4. Portal venous system is a valveless system

Ans. A: Femoral vein


Ref.: BDC 4th ed., vol.2, page-130

107. Normal portal pressure is 5-10 mm Hg (Schwartz)


Normal Portal pressure is 5-8 mm Hg (Kumar and Clark)

Portal system

 The portal vein drains blood from the small and large intestines, stomach, spleen, pancreas, and gallbladder.
 The superior mesenteric vein and the splenic vein unite behind the neck of the pancreas to form the portal vein.
 The portal trunk divides into 2 lobar veins.
 The right branch drains the cystic vein, and the left branch receives the umbilical and paraumbilical veins that
enlarge to form umbilical varices in portal hypertension.
 The coronary vein, which runs along the lesser curvature of the stomach, receives distal esophageal veins, which
also enlarge in portal hypertension.

Ans. A: 5-10 mm Hg
ANATOMY

Ref: Schwartz‖s Surgery, 9th ed., p-1111; Kumar and Clark‖s Clinical Medicine, 5th ed., p-163

108. The angle between the lower border of the 12th rib and the outer border of the erector spinae is known as the renal angle

Renal angle
 It is the angle between lateral border of Erector spinae and lower border of twelfth rib on the posterior aspect of
the trunk.
 In most of the cases any abnormality (e.g., pain, tenderness, fullness, bulge) in this region is an indicative of renal
origin.

Ans. A: 12th rib and lateral border of sacrospinalis


Ref: BDC, 4th ed., Vol.-II, p-301, 280 [Fig 23.2]; 5th ed., p-328

109. Between the 2 fascial layers (fatty/ superficial and deep/ Colle’s fascia) of the urogenital diaphragm lie deep transverse perineii;
superficial to the proximal urethral sphincter mechanism
Urogenital diaphragm

 It is a triangular musculo fascial diaphragm situated in the anterior part of perineum filling the gap of the pubic
arch.
 Components of urogenital diaphragm:

o Deep transverse perinei muscles

o Sphincter urethrae

o Superior/ superficial fascia of urogenital diaphragm


Anatomy 49

o Inferior/ deep fascia of urogenital diaphragm (Perineal membrane)


 Colles fascia does not form a part of urogenital diaphragm, but it is attached to the posterior border of urogenital
diaphragm

Ans. B: Superficial transverse perineii


Ref: BDC, 4th ed., p-332; 5th ed., p-358

110. Anteriorly, pouch of Douglas is bounded by the uterus and the posterior fornix of the vagina
Fornices of vagina

 The fornices of the vagina are the deepest portions of the vagina, extending into the recesses created by the
vaginal portion of cervix.
 There are three named fornices:
o The posterior fornix is the larger recess, behind the cervix. It is close to the rectouterine pouch.
o There are two smaller recesses in front and at the sides:
 The anterior fornix is close to the vesicouterine pouch.
 The lateral fornix.
 The fornices appear to be close to at least two erogenous zones, the AFE zone, which is near the anterior fornix,
and the cul-de-sac, which is near the posterior fornix

ANATOMY
Ans. B: Pouch of Douglas
Ref: BDC/II, 5th ed., p-254, 255

HEAD AND NECK

111. • Pharyngotympanic (Auditory) tube is 4 cm long, consists of two parts:


• Bony part forms posterior and lateral one third of the tube (12 cm long)
• Cartilaginous part forms the anterior and medial two third (25 cm long)
• Connects the middle ear with the nasopharynx
• Equalizes pressure on both sides of the tympanic membrane
• Usually closed to prevent entrance of particles from the nose
• Muscles that open auditory tube while swallowing, yawning and sneezing are levator veli palatini and tensor
veli palatini

Ans. D: Both A and B


Ref.: BDC 4th ed., vol.3, page-225

112. Branches of the External Carotid Artery


Superior thyroid artery
Ascending pharyngeal artery
Lingual artery
Facial artery
Occipital artery
Self Assessment & Review of FMGE / MCI Screening Examination
50

Posterior auricular artery


Superficial temporal artery
Maxillary artery

Ans. B: External carotid artery


Ref.: BDC 4th ed., vol.3, page-128, Clinical Anatomy-Snell, 8th ed., page-749

113. Ans. A: Cricothyroid


Ref.: BDC 4th ed., vol.3, page-244, Clinical Anatomy-Snell, 8th ed., page-767

114. Killians dehiscence is a weak part in the posterior wall of the pharynx which lies at the level of vocal folds or upper
border of the cricoid lamina and is limited inferiorly by the thick cricopharyngeal sphincter.

Pharyngeal diverticula results due to outpouching of the dehiscence


Two parts of the inferior constrictor has different nerve supplies, propulsive thyropharyngeus by the pharyngeal
plexus and the sphincter cricopharyngeus by the recurrent laryngeal nerve so there is possibility of neuromuscular
incoordination.
If cricopharyngeus fails to relax and the thyropharyngeus contracts bolus of food may be pushed backwards and tends
to produce a diverticulum

Ans. B: Lies in the anterior wall of pharynx


ANATOMY

Ref.: BDC 4th ed., vol.3, page-223, Clinical Anatomy-Snell, 8th ed., page-793

115. The parathyroid glands are ovoid bodies measuring about 6 mm long in their greatest diameter.
They are four in number and are closely related to the posterior border of the thyroid gland, lying within its fascial capsule.
The two superior parathyroid glands are the more constant in position and lie at the level of the middle of the
posterior border of the thyroid gland.
The two inferior parathyroid glands usually lie close to the inferior poles of the thyroid gland. They may lie within the
fascial sheath, embedded in the thyroid substance, or outside the fascial sheath

Ans. A: 4
Ref.: BDC 4th ed., vol.3, page-171, Clinical Anatomy-Snell, 8th ed., page-821

116. The nasolacrimal duct is about 0.5 in. (1.3 cm) long and emerges from the lower end of the lacrimal sac.
The duct descends downward, backward, and laterally in a bony canal and opens into the inferior meatus of the nose.
The opening is guarded by a fold of mucous membrane known as the lacrimal fold. This prevents air from being
forced up the duct into the lacrimal sac on blowing the nose.

Ans. D: Inferior meatus of nose


Ref.: BDC 4th ed., vol.3, page-63, Clinical Anatomy-Snell, 8th ed., page-694

117. Ans. A: Inferior thyroid artery


Ref.: BDC 4th ed., vol.3, page-128,175, Clinical Anatomy-Snell, 8th ed., page-749
Anatomy 51

118. • The muscles of the face are embedded in the superficial fascia, and most arise from the bones of the skull and are
inserted into the skin.
• The orifices of the face, namely, the orbit, nose, and mouth, are guarded by the eyelids, nostrils, and lips,
respectively.
• It is the function of the facial muscles to serve as sphincters or dilators of these structures.
• A secondary function of the facial muscles is to modify the expression of the face.
• All the muscles of the face are developed from the second pharyngeal arch and are supplied by the facial nerve.

Ans. C: Develops from 3rd pharyngeal arch


Ref.: BDC 4th ed., vol.3, page-50, Clinical Anatomy-Snell, 8th ed., page-731

119. All the intrinsic and the extrinsic muscles, except the palatoglossus are supplied by the hypoglossal nerve.
The palatoglossus is supplied by the cranial root of the accessory nerve through the pharyngeal plexus

Lingual nerve is the nerve of general sensation and the chorda tympani is the nerve of taste for the anterior two
thirds of the tongue except vallate papillae

Ans. A: Hypoglossal nerve


Ref.: BDC 4th ed., vol.3, page-252

ANATOMY
120. Ans. A: Stensons duct
Ref.: BDC 4th ed., vol.3, page-136, Clinical Anatomy-Snell, 8th ed., page-787

121. The parotid gland lies in a deep hollow below the external auditory meatus, behind the ramus of the mandible, and in
front of the sternocleidomastoid muscle.

The facial nerve divides the gland into superficial and deep lobes.

The parotid duct, or Stenson duct, is about 2 in. (5 cm) long and passes forward across the masseter about a
fingerbreadth below the zygomatic arch.

It passes through the buccal fat, buccopharyngeal fascia, and buccinator muscle then opens into the vestibule of
the mouth next to the maxillary second molar tooth. The buccinator acts as a valve that prevents inflation of the duct
during blowing.

The submandibular glands are a pair of glands located beneath the lower jaws, superior to the digastric muscles.
The secretion produced enters the oral cavity via Wharton‖s ducts. Approximately 70% of saliva in the oral cavity is
produced by the submandibular glands, even though they are much smaller than the parotid glands.

Ans. D: Masesster
Ref.: BDC 4th ed., vol.3, page-136, Clinical Anatomy-Snell, 8th ed., page-787

122. The sclera is much thicker behind than in front; the thickness of its posterior part at the macula is 1 mm.

The sclera thins to 0.3 mm just behind the recti muscle insertions (about 6 mm behind the corneoscleral junction)
and this area is extremely vulnerable to traumatic rupture. In fact this is the most common site of a ruptured globe due
to blunt trauma.
Self Assessment & Review of FMGE / MCI Screening Examination
52

At the equator the sclera measures 0.4-0.5 mm in thickness.

It is thickest behind, near the entrance of the optic nerve. However it is weakest at the entrance of theoptic nerve.

Ans. D: At the insertion of recti muscles


Ref.: BDC 4th ed., vol.3, page-270

123. Recurrent Laryngeal Nerve paralysis:


• If both recurrent laryngeal nerves are interrupted, the vocal cords lie in a cadaveric position in between
abduction and adduction and phonation is completely lost.
• When only one recurrent laryngeal nerve is affected, the opposite vocal cord compensates for it and phonation is
possible but there is hoarseness of voice
Superior Laryngeal Nerve paralysis
It divides into external and internal laryngeal nerves. External laryngeal nerve supplies cricothyroid and inferior
constrictor and internal laryngeal nerve supplies mucous membrane of the larynx upto the level of vocal folds:
• Asymmetric vocal cord tension
• Produces diplophonia
• Loss of vocal fold tension (lowers pitch of voice)
• Inaccurate vocal cord apposition
• Paralysed side slightly shortened and bowed
ANATOMY

• May be depressed below level of normal side


• Rotation of AP axis of vocal cords
• Posterior commissure points to side of paralysis
• Loss of laryngeal sensation and increased risk of aspiration

Ans. B: Both recurrent laryngeal nerve palsy


Ref.: BDC 4th ed., vol.3, page-247, Clinical Anatomy-Snell, 8th ed., page-806

124. The annulus of Zinn, also known as the annular tendon or common tendinous ring, is a ring of fibrous tissue
surrounding the optic nerve at its entrance at the apex of the orbit.
It can be used to divide the regions of the superior orbital fissure.
The arteries surrounding the optic nerve are sometimes called the “circle of Zinn-Haller” (CZH).
Some sources distinguish between these terms more precisely, with the annulus tendineus communis being the parent
structure, divided into two parts:
i. A lower, the ligament or tendon of Zinn, which gives origin to the Rectus inferior, part of the Rectus internus,
and the lower head of origin of the Rectus lateralis.
ii. An upper, which gives origin to the Rectus superior, the rest of the Rectus medialis, and the upper head of the
Rectus lateralis. This upper band is sometimes termed the superior tendon of Lockwood.

The site of origin of the superior oblique muscle is from the lesser wing of sphenoid above the optic canal.

Ans. D: Superior oblique


Ref.: BDC 4th ed., vol.3, page-22, Clinical Anatomy-Snell, 8th ed., page-694
Anatomy 53

125. Depression of the Mandible


Depression of the mandible is brought about by contraction of the digastrics, the geniohyoids, and the mylohyoids; the
lateral pterygoids play an important role by pulling the mandible forward.
Elevation of the Mandible
Elevation of the mandible is brought about by contraction of the temporalis, the masseter, and the medial
pterygoids. The head of the mandible is pulled backward by the posterior fibers of the temporalis.
Protrusion of the Mandible
In protrusion, the lower teeth are drawn forward over the upper teeth, which is brought about by contraction of the
lateral pterygoid muscles of both sides, assisted by both medial pterygoids.
Retraction of the Mandible
The articular disc and the head of the mandible are pulled backward into the mandibular fossa. Retraction is brought
about by contraction of the posterior fibers of the temporalis.
Lateral Chewing Movements
These are accomplished by alternately protruding and retracting the mandible on each side. For this to take place, a
certain amount of rotation occurs, and the muscles responsible on both sides work alternately like turning the chin to
left side produced by left lateral pterygoid and right medial pterygoid and vice versa.

Ans. C: Lateral pterygoids


Ref.: BDC 4th ed., vol.3, page-152, Clinical Anatomy-Snell, 8th ed., page-720

ANATOMY
126. Located posteriorly between the greater and lesser wings of the sphenoid; Superior orbital fissure communicates with
the middle cranial fossa.

It transmits the lacrimal nerve, the frontal nerve, the trochlear nerve, the oculomotor nerve (upper and lower
divisions), the abducent nerve, the nasociliary nerve, and the superior ophthalmic vein.

Zygomatic nerve passes through inferior orbital fissure

Ans. D: Zygomatic nerve


Ref.: BDC 4th ed., vol.3, page-108,28, Clinical Anatomy-Snell, 8th ed., page-696

127. Superior Meatus


The superior meatus lies below the superior concha. It receives the openings of the posterior ethmoid sinuses.
Middle Meatus
The middle meatus lies below the middle concha. It has a rounded swelling called the bulla ethmoidalis that is formed
by the middle ethmoidal air sinuses, which open on its upper border.
A curved opening, the hiatus semilunaris, lies just below the bulla. The maxillary sinus opens into the middle meatus
through the hiatus semilunaris.
The opening of the frontal air sinus is seen in the anterior part of the hiatus semilunaris
Inferior Meatus
The inferior meatus is below and lateral to the inferior nasal concha; the nasolacrimal duct opens into this meatus
under cover of the anterior part of the inferior concha.

Ans. C: Posterior ethmoid sinuses


Ref.: BDC 4th ed., vol.3, page-231, Clinical Anatomy-Snell, 8th ed., page-797
Self Assessment & Review of FMGE / MCI Screening Examination
54

128. Ophthalmic artery is the branch of cerebral part of internal carotid artery

Ans. D: Ophthalmic artery


Ref.: BDC 4th ed., vol.3, page-128,103, Clinical Anatomy-Snell, 8th ed., page-749

129. Facial Nerve descends in the posterior wall of the middle ear, behind the pyramid, and emerges through the
stylomastoid foramen into the neck.
The greater petrosal nerve arises from the facial nerve at the geniculate ganglion.
It contains secretomotor (parasympathetic) fibers to the lacrimal gland, submandibular and sublingual salivary
glands, and the glands of the nose,the palate and the pharynx
The nerve is joined by the deep petrosal nerve from the sympathetic plexus and forms the nerve of the pterygoid canal
which ends in the pterygopalatine ganglion.
The nerve to the stapedius arises from the facial nerve which supplies the muscle within the pyramid.
The chorda tympani arises from the facial nerve just above the stylomastoid foramen. The nerve leaves the middle ear
through the petrotympanic fissure and enters the infratemporal fossa, where it joins the lingual nerve.
The chorda tympani contains taste fibers from the mucous membrane covering the anterior two thirds of the tongue
(not the vallate papillae) and the floor of the mouth.

Ans. D: Swallowing
ANATOMY

Ref.: BDC 4th ed., vol.3, page-138, Clinical Anatomy-Snell, 8th ed., page-712

130. The first plane is the venous plane and consists of the retromandibular vein and its tributaries and branches
Deep to venous plane is the important nervous plane. The importance of this plane is the presence of the facial
(VII) nerve. The facial nerve leaves the skull through the stylomastoid foramen and immediately enters the deep part
of the parotid gland where it gives off its branches:
i. Posterior auricular
ii. Motor branch to posterior belly of digastric
iii. Temporal branch
iv. Zygomatic branch
v. Buccal branches
vi. Mandibular branch
vii. Cervical branch

Deep to the nerves lies the arterial plane which includes terminal parts of the external carotid artery and its
branches:
i. External carotid artery
ii. Occipital artery
iii. Maxillary artery
iv. Transverse facial artery
v. Superficial temporal artery

The deepest part of the parotid region is the parotid bed and houses the deep part of the gland which fills the small
space between the neck of the condyle of the mandible and the mastoid process. Other structures forming the floor of
this space are the:
Anatomy 55

i. Styloid process
ii. Stylohyoid muscle
iii. Stylopharyngeus muscle
iv. Posterior belly of the digastric muscle

Ans. A: Facial artery


Ref.: BDC 4th ed., vol.3, page-136, Clinical Anatomy-Snell, 8th ed., page-787

131. Submandibular Gland lies beneath the lower border of the body of the mandible and is divided into superficial and
deep parts by the mylohyoid muscle.
The deep part of the gland lies beneath the mucous membrane of the mouth on the side of the tongue.
The submandibular duct emerges from the anterior end of the deep part of the gland and runs forward beneath the
mucous membrane of the mouth. It opens into the mouth on a small papilla, which is situated at the side of the
frenulum of the tongue.
Parasympathetic secretomotor supply is from the facial nerve via the chorda tympani, and the submandibular
ganglion.

Ans. C: Facial
Ref.: BDC 4th ed., vol.3, page-161, Clinical Anatomy-Snell, 8th ed., page-789

ANATOMY
132. The posterior triangle is bounded posteriorly by the trapezius muscle, anteriorly by the sternocleidomastoid muscle,
and inferiorly by the clavicle.
The posterior cervical triangle is subdivided into the following triangles by the inferior belly of the omohyoid muscle:

Occipital triangle, whose contents are:


a. Nerve to rhombideus
b. Cutaneous branches of cervical plexus of nerves
c. Spinal accessory nerve
d. Upper part of brachial plexus
e. Transverse cervical artery and vein
f. Cervical lymph nodes
g. Occipital artery
h. Supraclavicular nodes

Subclavian triangle (also known as omoclavicular), whose contents are:


a. Third part of subclavian artery
b. Part of subclavian vein
c. Suprascapular artery and vein
d. Supraclavicular lymph nodes
e. Three trunks of brachial plexus
f. Nerve to serratus anterior
g. Nerve to subclavius
h. Suprascapular nerve
Self Assessment & Review of FMGE / MCI Screening Examination
56

i. Lower part of external jugular vein


j. Transverse cervical artery and vein

Ans. C: Internal jugular vein


Ref.: BDC 4th ed., vol.3, page-72, Clinical Anatomy-Snell, 8th ed., page-747

133. Single/pure movements-produced as a result of combined actions of muscles.


1. Upward rotation/elevation-superior rectus and inferior oblique
2. Downward rotation/depression-inferior rectus and superior oblique
3. Medial rotation/adduction-superior rectus, medial rectus and inferior rectus
4. Lateral rotation/abduction-superior rectus, lateral rectus and inferior rectus
5. Intortion-superior oblique and superior rectus
6. Extortion-inferior oblique and inferior rectus

Ans. C: Both of the above


Ref.: BDC 4th ed., vol.3, page-110, Clinical Anatomy-Snell, 8th ed., page-694

134. The external auditory meatus is 4 cm long and conducts sound waves from the auricle to the tympanic membrane.

The framework of the anterior and medial two-thirds of the meatus is elastic cartilage (measures 25 mm in length),
and the posterior and lateral one third is bony, formed by the tympanic plate (measures 12 mm in length).
ANATOMY

The sensory nerve supply of the lining skin is derived from the auriculotemporal nerve and the auricular branch of the
vagus nerve.

The lymph drainage is to the superficial parotid, mastoid, and superficial cervical lymph nodes.

Ans. C: 25 mm
Ref.: BDC 4th ed., vol.3, page-225

135. The lingual nerve passes forward into the submandibular region from the infratemporal fossa by running beneath the
origin of the superior constrictor muscle, which is attached to the posterior border of the mylohyoid line on the
mandible. Here, it is closely related to the last molar tooth and is liable to be damaged in cases of clumsy extraction of
an impacted third molar.

Ans. C: Lingual nerve


Ref.: BDC 4th ed., vol.3, page-156, Clinical Anatomy-Snell, 8th ed., page-762

136. Ans. B: Upper 2nd molar


Ref.: BDC 4th ed., vol.3, page-137, Clinical Anatomy-Snell, 8th ed., page-787

137. The primary action of the superior oblique muscle is intorsion (internal rotation), the secondary action is depression
(primarily in the adducted position) and the tertiary action is abduction.

Ans. C: Intortion, abduction and depression


Ref.: BDC 4th ed., vol.3, page-109, Clinical Anatomy-Snell, 8th ed., page-694
Anatomy 57

138. Genioglossus is the fan-shaped extrinsic tongue muscle that forms the majority of the body of the tongue.

Its origin is the mental spine of the mandible and its insertions are the hyoid bone and the dorsum of the tongue.

Innervated by the hypoglossal nerve (CN XII), it depresses and protrudes the tongue

Contraction of the genioglossus stabilizes and enlarges the portion of the upper airway that is most vulnerable to
collapse.

A relaxation of the genioglossus and geniohyoideus muscles, especially during REM sleep, is implicated in
Obstructive Sleep Apnea (OSA.)

Peripheral damage to the hypoglossal nerve can result in deviation of the tongue to the damaged side

Ans. B: Genioglossus
Ref.: Gray‖s anatomy 38th ed. Page-1725, BDC 4th ed., vol.3, page-252

139. The trigeminal nerve through its three branches is the chief sensory nerve of the face.
The skin over the angle of the jaw and over the parotid gland is supplied by the great auricular nerve (C2,C3)

Ans. A: Great auricular nerve


Ref.: BDC 4th ed., vol.3, page-54

ANATOMY
140. Ans. B: Posterior cricoarytenoid
Ref.: BDC 4th ed., vol.3, page-245

141. Adductor of vocal cords


i. Thyroarytenoid muscle:
R and L muscles; attached to thyroid and arytenoid cartilages on each side.
Action shortens and relaxes vocal ligament.
Note: deeper inner fibers referred to as “vocalis muscle”.
ii. Lateral cricoarytenoid muscle: (R and L muscles):
Attached to cricoid and arytenoid cartilage on each side.
Closes or adducts vocal folds.
Supplied by Recurrent laryngeal nerve.
iii. Cricothyroid muscle:
Attached to cricoid and thyroid cartilages.
Tilts the thyroid cartilage, thus increasing tension of vocal folds
Supplied by external laryngeal nerve
iv. Inter-arytenoid muscle (transverse and oblique)
Attached between right and left arytenoid cartilages
Closes inlet of larynx
Supplied by Recurrent laryngeal nerve
Self Assessment & Review of FMGE / MCI Screening Examination
58

Abductor of vocal cords


i. Posterior cricoarytenoid muscle
Attached to cricoid and arytenoid cartilages
Move arytenoid cartilages so as to move both vocal folds apart, “open” of abduct vocal folds
Supplied by Recurrent laryngeal nerve

Vocalis muscle (derived from inner and deeper fibers of thyroarytenoid msucle)

Alters vocal fold tension/relaxation during speaking or singing

Supplied by Recurrent laryngeal nerve

All intrinsic muscles of the larynx are supplied by the recurrent laryngeal nerve except for cricothyroid which is
supplied by external laryngeal nerve.

Ans: C: Cricothyroid
Ref.: BDC 4th ed., vol.3, page-244, Clinical Anatomy-Snell, 8th ed., page-806

142. Ans. C: Parotid gland


Ref.: BDC 4th ed., vol.3, page-133,137, Clinical Anatomy-Snell, 8th ed., page-763

143. Most cases of congenital torticollis are a result of excessive stretching of the sternocleidomastoid muscle during a
ANATOMY

difficult labor.

Hemorrhage occurs into the muscle and may be detected as a small, rounded during the early weeks after birth. Later,
this becomes invaded by fibrous tissue, which contracts and shortens the muscle.

The mastoid process is thus pulled down toward the sternoclavicular joint of the same side, the cervical spine is flexed,
and the face looks upward to the opposite side.

If left untreated, asymmetrical growth changes occur in the face, and the cervical vertebrae may become wedge
shaped.

Spasmodic Torticollis
Spasmodic torticollis, which results from repeated chronic contractions of the sternocleidomastoid and trapezius
muscles, is usually psychogenic in origin. Section of the spinal part of the accessory nerve may be necessary in severe cases.

Ans. B: Sternocleidomastoid
Ref.: BDC 4th ed., vol.3, page-74, Clinical Anatomy-Snell, 8th ed., page-742

144. A conical projection called the pyramid lies near the junction of the posterior and medial walls of the middle ear.
It has an opening at its apex for the passage of the tendon of the stapedius muscle

Ans. B: Stapedius
Ref.: BDC 4th ed., vol.3, page-260

145. Their secretions of submandibular gland, like the secretions of other salivary glands, are regulated directly by the
parasympathetic nervous system and indirectly by the sympathetic nervous system.
Anatomy 59

i. Parasympathetic innervation to the submandibular glands is provided by the superior salivatory nucleus via the
chorda tympani, a branch of the facial nerve that synapses in the submandibular ganglion after which it follows
the Lingual nerve leaving this nerve as it approaches the gland. Increased parasympathetic activity promotes the
secretion of saliva.

ii. The sympathetic nervous system regulates submandibular secretions through vasoconstriction of the arteries that
supply it. Increased sympathetic activity reduces glandular blood flow, thereby decreasing salivary secretions and
producing an enzyme rich mucous saliva.

Ans. D: Auriculotemporal nerve


Ref.: BDC’s Anatomy, Vol-III, 4th ed.,p-163

146. Muscle which abduct the vocal cord is posterior cricoarytenoid only

Ans. D: Posterior Cricoarytenoid


Ref.: BDC, 4th ed., Vol.-III, p-243; 5th ed., p-242

147. Main source of arterial supply to tonsil is tonsilar branch of facial artery
Additional sources are ascending palatine branch of the facial artery, dorsal lingual branch of the lingual artery, ascending
pharyngeal branch of the external carotid artery and greater palatine branch of the maxillary artery

ANATOMY
Ans. C: Superior thyroid artery
Ref.: BDC, 5th ed., Vol.-III, p-136, 216

148. Glossopharyngeal and lesser palatine nerves supply palatine tonsil

Palatine tonsil
 The Palatine Tonsils are two prominent masses situated one on either side between the glossopalatine and
pharyngopalatine arches.
 Each tonsil consists fundamentally of an aggregation of lymphoid tissue underlying the mucous membrane
between the palatine arches.
 In the child the tonsils are relatively (and frequently absolutely) larger than in the adult
 The follicles of the tonsil are lined by a continuation of the mucous membrane of the pharynx, covered with
stratified squamous epithelium
 Arteries supplying the tonsil are the:
o Dorsalis linguae from the lingual
o The ascending palatine and tonsillar from the external maxillary
o The ascending pharyngeal from the external carotid
o The descending palatine branch of the internal maxillary
o A twig from the small meningeal.
 The veins end in the tonsillar plexus, on the lateral side of the tonsil
 The nerves are derived from the sphenopalatine ganglion, and from the glossopharyngeal.

Ans. C: Glossopharyngeal nerve


Ref.: BDC, 4th ed., Vol.-III, p-218; 5th ed., p-216
Self Assessment & Review of FMGE / MCI Screening Examination
60

149. The Glossopharyngeal nerve is the nerve for both general sensation and taste sensation for the posterior 1/3rd of the tongue
including the circumvallate papillae
Nerve supply of tongue
 Motor supply: All the intrinsic muscles, except the palatoglossus are supplied by the hypoglossal nerve
(The palatoglossus is supplied by the cranial part of accessory nerve through the pharyngeal plexus)
 Sensory supply :
o Anterior 2/3rd of the tongue - Lingual nerve is the nerve of general sensation and the chorda tympani is
the nerve of taste for the anterior two-thirds of the tongue.
o Posterior 1/3rd of tongue - The glossopharyngeal nerve is the nerve for both general sensation and taste
sensation for the posterior 1/3rd of the tongue.
 The posterior most part of the tongue is supplied by the vagus nerve through the internal laryngeal branch

Ans. C: Glossopharyngeal nerve


Ref.: BDC, 5th ed., p-253

150. Internal jugular vein is the direct continuation of the sigmoid sinus

The sigmoid sinuses


 They are two areas beneath the brain which allow blood to drain inferiorly from the posterior center of the head.
 They drain from the transverse sinuses and converge with the inferior petrosal sinuses to form the internal
jugular vein
ANATOMY

 Each sigmoid sinus begins beneath the temporal bone and follows a tortuous course to the jugular foramen, at
which point the sinus becomes continuous with the internal jugular vein

The internal jugular vein


 It collects the blood from the brain, from the superficial parts of the face, and from the neck.
 It is directly continuous with the transverse sinus, and begins in the posterior compartment of the jugular
foramen, at the base of the skull.
 This vein receives in its course the inferior petrosal sinus, the common facial, lingual, pharyngeal, superior and
middle thyroid veins, and sometimes the occipital.
 The thoracic duct on the left side and the right lymphatic duct on the right side open into the angle of union of the
internal jugular and subclavian veins.

The external jugular vein


 It receives the greater part of the blood from the exterior of the cranium and the deep parts of the face
 It is formed by the junction of the posterior division of the posterior facial with the posterior auricular vein.
 It commences in the substance of the parotid gland, on a level with the angle of the mandible
 This vein receives the occipital occasionally, the posterior external jugular, and, near its termination, the
transverse cervical, transverse scapular, and anterior jugular veins; in the substance of the parotid, a large branch
of communication from the internal jugular joins it.

The anterior jugular vein


 It begins near the hyoid bone by the confluence of several superficial veins from the submaxillary region.
 It descends between the median line and the anterior border of the Sternocleidomastoideus, and, at the lower part
of the neck, passes beneath that muscle to open into the termination of the external jugular, or, in some instances,
into the subclavian vein

Ans. B: Sigmoid sinus


Ref.: BDC/III, 5th ed., p-183
Anatomy 61

151. Vestibulo-cochlear nerve comprises of hearing and vestibular parts


Scala tympani
 It is one of the perilymph-filled cavities in the cochlear labyrinth of the ear.
 It is separated from the scala media by the basilar membrane, and it extends from the round window to the
helicotrema, where it continues as scala vestibuli.
 The purpose of the perilymph-filled scala tympani and scala vestibuli is to transduce the movement of air that
causes the tympanic membrane and the ossicles to vibrate, to movement of liquid and the basilar membrane.
 This movement is conveyed to the organ of Corti inside the scala media, composed of hair cells attached to the
basilar membrane and their stereocilia embedded in the tectorial membrane.
 The movement of the basilar membrane compared to the tectorial membrane causes the sterocilia to bend.
 They then depolarise and send impulses to the brain via the cochlear nerve.
 This produces the sensation of sound.

Ans. C: Vestibulocochlear nerve


Ref.: Gray‖s Anatomy for students, 1st ed., p-869; BDC/III, 5th ed., p-267f, 359f

152. Trochlear nerve ends by supplying the superior oblique muscle on its orbital surface
Remember
 SO-4, LR-6, Rest by 3
 Superior oblique by 4th cranial nerve
 Lateral rectus by 6th cranial nerve
 Rest of the muscles by 3rd cranial nerve

ANATOMY
Ans. B: Superior oblique
Ref.: BDC, 4th ed., Vol-III, p-108, 109; 5th ed., p-350

BRAIN AND SPINAL CORD

153. Numerous arteries supply the dura mater from the internal carotid, maxillary, ascending pharyngeal, occipital, and
vertebral arteries.
From a clinical standpoint, the most important is the middle meningeal artery, which is commonly damaged in head
injuries.
The middle meningeal artery arises from the maxillary artery in the infratemporal fossa. To enter the cranial cavity, it
passes through the foramen spinosum to lie between the meningeal and endosteal layers of dura.
The anterior (frontal) branch‖s course corresponds roughly to the line of the underlying precentral gyrus of the brain.
The posterior (parietal) branch curves backward and supplies the posterior part of the dura mater.

Ans. D: Basilar artery


Ref.: BDC 4th ed., vol.3, page-92, Clinical Anatomy-Snell, 8th ed., page-682

154. The middle cerebral artery is the largest terminal branch of the internal carotid artery and it runs laterally in the lateral
cerebral sulcus of the brain.
It supplies the entire lateral surface of the cerebral hemisphere except the narrow strip along the superomedial
border extending from the frontal pole to the parieto-occipital sulcus(which is supplied by the anterior cerebral artery)
and the occipital pole and inferior temporal gyrus (both of which are supplied by the posterior cerebral artery).
Anterior cerebral artery chiefly supplies the medial surface

Ans. A: Middle cerebral artery


Ref.: BDC 4th ed., vol.3, page-386, Clinical Anatomy-Snell, 8th ed., page-750
Self Assessment & Review of FMGE / MCI Screening Examination
62

155. The spinal cord is a cylindrical, grayish white structure that begins above at the foramen magnum, where it is
continuous with the medulla oblongata of the brain.
It terminates below in the adult at the level of the lower border of the first lumbar vertebra.
In the young child, it is relatively longer and ends at the upper border of the third lumbar vertebra.
The spinal cord in the cervical region gives origin to the brachial plexus, and in the lower thoracic and lumbar regions,
where it gives origin to the lumbosacral plexus.
Inferiorly, the spinal cord tapers off into the conus medullaris, from the apex of which a prolongation of the pia mater,
the filum terminale, descends to be attached to the back of the coccyx.

Ans. C: L1
Ref.: BDC 4th ed., vol.3, page-309, Clinical Anatomy-Snell, 8th ed., page-867

156. Ans. C: 12th nerve


Ref.: BDC 4th ed., vol.3, page-331, Clinical Anatomy-Snell, 8th ed., page-769

157. Cranial Nerve I: Olfactory


 Arises from the olfactory epithelium.
 Passes through the cribriform plate of the ethmoid bone.
 Fibers run through the olfactory bulb and terminate in the primary olfactory cortex.
Functions solely by carrying afferent impulses for the sense of smell.
ANATOMY


Cranial Nerve II: Optic
 Arises from the retina of the eye.
 Optic nerves pass through the optic canals and converge at the optic chiasm.
 They continue to the thalamus where they synapse.
 From there, the optic radiation fibers run to the visual cortex.
 Functions solely by carrying afferent impulses for vision.
Cranial Nerve III: Oculomotor
 Fibers extend from the ventral midbrain, pass through the superior orbital fissure, and go to the extrinsic
eye muscles.
 Functions in raising the eyelid, directing the eyeball, constricting the iris, and controlling lens shape.
 Parasympathetic cell bodies are in the ciliary ganglia.
Cranial Nerve IV: Trochlear
 Fibers emerge from the dorsal midbrain and enter the orbits via the superior orbital fissures; innervate the
superior oblique muscle.
 Primarily a motor nerve that directs the eyeball.
Cranial Nerve V: Trigeminal
 Three divisions: ophthalmic (V1), maxillary (V2), and mandibular (V3).
 Fibers run from the face to the pons via the superior orbital fissure (V1), the foramen rotundum (V2), and the
foramen ovale (V3).
 Conveys sensory impulses from various areas of the face (V1) and (V2), and supplies motor fibers (V3) for
mastication.
Anatomy 63

Cranial Nerve VI: Abdcuens


 Fibers leave the inferior pons and enter the orbit via the superior orbital fissure.
 Primarily a motor nerve innervating the lateral rectus muscle.
Cranial Nerve VII: Facial
 Fibers leave the pons, travel through the internal acoustic meatus, and emerge through the stylomastoid
foramen to the lateral aspect of the face.
 Mixed nerve with five major branches.
 Motor functions include facial expression, and the transmittal of autonomic impulses to lacrimal and salivary
glands.
 Sensory function is taste from the anterior two-thirds of the tongue.
Cranial Nerve VIII: Vestibulocochlear
 Fibers arise from the hearing and equilibrium apparatus of the inner ear, pass through the internal acoustic
meatus, and enter the brainstem at the pons-medulla border.
 Two divisions – cochlear (hearing) and vestibular (balance).
 Functions are solely sensory – equilibrium and hearing.
Cranial Nerve IX: Glossopharyngeal
 Fibers emerge from the medulla, leave the skull via the jugular foramen, and run to the throat.
 Nerve IX is a mixed nerve with motor and sensory functions.

ANATOMY
 Motor – innervates part of the tongue and pharynx and provides motor fibers to the parotid salivary gland.
 Sensory – fibers conduct taste and general sensory impulses from the tongue and pharynx.
Cranial Nerve X: Vagus
 The only cranial nerve that extends beyond the head and neck.
 Fibers emerge from the medulla via the jugular foramen.
 The vagus is a mixed nerve.
 Most motor fibers are parasympathetic fibers to the heart, lungs, and visceral organs.
 Its sensory function is in taste.
Cranial Nerve XI: Accessory
 Formed from a cranial root emerging from the medulla and a spinal root arising from the superior region of the
spinal cord.
 The spinal root passes upward into the cranium via the foramen magnum.
 The accessory nerve leaves the cranium via the jugular foramen.
 Primarily a motor nerve.
 Supplies fibers to the larynx, pharynx, and soft palate.
 Innervates the trapezius and sternocleidomastoid, which move the head and neck.
Cranial Nerve XII: Hypoglossal
 Fibers arise from the medulla and exit the skull via the hypoglossal canal.
 Innervates both extrinsic and intrinsic muscles of the tongue, which contribute to swallowing and speech.

Ans. A: Optic nerve


Ref.: BDC 4th ed., vol.3, page-109, Clinical Anatomy-Snell, 8th ed., page-694
Self Assessment & Review of FMGE / MCI Screening Examination
64

158. Ans. B: Trigeminal


Ref.: Clinical Anatomy-Snell, 8th ed., page-759

159. Tributaries of cavernous sinus


1. From the orbit:
i. Superior ophthalmic veins
ii. Inferior ophthalmic vein or its branch
iii. Central vein of the retina
2. From the brain:
i. Superficial middle cerebral vein
ii. Inferior cerebral veins
3. From the meninges
i. Sphenoparietal sinus
ii. Frontal trunk of the middle meningeal vein
Draining channels
i. Transverse sinus through the superior petrosal sinus
ii. Internal jugular vein through inferior petrosal sinus and through a plexus around the internal carotid artery
iii. Into pterygoid plaexus of veins through the emissary veins
iv. Facial vein through the superor ophthalmic vein

Ans. A: Superior petrosal sinus


Ref.: BDC 4th ed., vol.3, page-94
ANATOMY

160. The amygdala (almond shaped) is part of the basal ganglia, and is situated in the temporal lobe. It has many
connections with other parts of the brain, most particularly with the limbic system, for which reason it is considered
part of the limbic system (although not part of the limbic lobe itself). The amygdala is critically involved in computing
the emotional significance of events.

The raphe nuclei are a moderate-size cluster of nuclei found in the brain stem. Their main function is to release
serotonin to the rest of the brain.

The Dentate nucleus is located within the deep white matter of each cerebellar hemisphere. It is the largest of the
four deep cerebellar nuclei, the others being the fastigial nucleus and the interposed nucleus (globose and emboliform
nuclei combined). It is responsible for the planning, initiation and control of volitional movements.

The red nucleus is a structure in the rostral midbrain involved in motor coordination.

Ans. A: Amygdala
Ref.: BDC 4th ed., vol.3, page-361,329, Clinical Anatomy-Snell, 8th ed., page-687

161. The midbrain is the narrow part of the brain that passes through the tentorial notch and connects the forebrain to the
hindbrain.
The midbrain comprises two lateral halves called the cerebral peduncles; each of these is divided into an anterior part,
the crus cerebri, and a posterior part, the tegmentum, by a pigmented band of gray matter, the substantia nigra.
The narrow cavity of the midbrain is the cerebral aqueduct, which connects the third and fourth ventricles.
The tectum is the part of the midbrain posterior to the cerebral aqueduct; it has four small surface swellings, namely,
the two superior and two inferior colliculi.
Cross section of midbrain at the level of superior colliculus:
Grey matter
i. Central grey matter contains nucleus of oculomotor nerve and the mesencephalic nucleus of the trigeminal nerve
ii. Pretectal nucleus
Anatomy 65

iii. Red nucleus


iv. Substantia nigra
v. Superior colliculus
Nucleus of trochlear nerve is present in the midbrain at the level of inferior colliculus

Ans. D: Red nucleus and occulomotor nerve nucleus


Ref.: BDC 4th ed., vol.3, page-329, Clinical Anatomy-Snell, 8th ed., page-689

162. The spinal root arises from nerve cells in the anterior gray column (horn) of the upper five segments of the cervical
part of the spinal cord. The nerve ascends alongside the spinal cord and enters the skull through the foramen
magnum.
It then turns laterally to join the cranial root.
The two roots unite and leave the skull through the jugular foramen.
The roots then separate:
The cranial root joins the vagus nerves and is distributed in its branches to the muscles of the soft palate and pharynx
(via the pharyngeal plexus) and to the muscles of the larynx (except the cricothyroid muscle).
The spinal root runs downward and laterally and enters the deep surface of the sternocleidomastoid muscle, which it
supplies, and then crosses the posterior triangle of the neck to supply the trapezius muscle.
The accessory nerve thus brings about movements of the soft palate, pharynx, and larynx and controls the movements
of the sternocleidomastoid and trapezius muscles.

ANATOMY
Ans. C: Sternocleidomastoid
Ref.: BDC 4th ed., vol.3, page-335, Clinical Anatomy-Snell, 8th ed., page-767

163. The cerebrum is the largest part of the brain and consists of two cerebral hemispheres connected corpus callosum.
The surface layer of each hemisphere is called the cortex and is composed of gray matter.
The frontal lobe is situated in front of the central sulcus and above the lateral sulcus.
The parietal lobe is situated behind the central sulcus and above the lateral sulcus. The occipital lobe lies below the
parieto-occipital sulcus.
Below the lateral sulcus is situated the temporal lobe.
The precentral gyrus lies immediately anterior to the central sulcus and is known as the motor area and control
voluntary movements on the opposite side of the body.
Most nerve fibers cross over to the opposite side in the medulla oblongata as they descend to the spinal cord.
In the motor area, the body is represented in an inverted position.
The postcentral gyrus lies immediately posterior to the central sulcus and is known as the sensory area. The small
nerve cells in this area receive and interpret sensations of pain, temperature, touch, and pressure from the opposite
side of the body.
The superior temporal gyrus lies immediately below the lateral sulcus. The middle of this gyrus is concerned with the
reception and interpretation of sound and is known as the auditory area.
Broca‖s area, or the motor speech area, lies just above the lateral sulcus. It controls the movements employed in speech.
It is dominant in the left hemisphere in right-handed persons and in the right hemisphere in left-handed persons.
The visual area is situated on the posterior pole and medial aspect of the cerebral hemisphere in the region of the
calcarine sulcus. It is the receiving area for visual impressions.

Ans. C: Superior temporal gyrus


Ref.: BDC 4th ed., vol.3, page-351, Clinical Anatomy-Snell, 8th ed., page-687
Self Assessment & Review of FMGE / MCI Screening Examination
66

164. The facial muscles are innervated by the facial nerve.


Damage to the facial nerve in the internal acoustic meatus (by a tumor), in the middle ear (by infection or operation),
in the facial nerve canal (perineuritis, Bell‖s palsy), or in the parotid gland (by a tumor) or caused by lacerations of the
face will cause distortion of the face, with drooping of the lower eyelid, and the angle of the mouth will sag on the
affected side. This is essentially a lower motor neuron lesion.
Check voluntary movement of the upper part of the face on the affected side: in supranuclear lesions such as a
cortical stroke (upper motor neuron; above the facial nucleus in the pons), the upper third of the face is spared
while the lower two thirds are paralyzed.
The orbicularis, frontalis, and corrugator muscles are innervated bilaterally, which explains the pattern of facial
paralysis in these cases

Ans. D: Upper half of the face normal, lower half affected


Ref.: Clinical Anatomy-Snell, 8th ed., page-732

165. Nucleus Ambiguus


Function:
Motor innervation of ipsilateral muscles of the soft palate, pharynx, larynx and upper esophagus.
Pathway:
Axons of motor neurons in the nucleus ambiguus course with three cranial nerves: C.N. IX (glossopharyngeal),
C.N. X (vagus), C.N. XI (rostral or cranial portion of spinoaccessory) to innervate striated muscles of the soft palate,
pharynx, larynx and upper esophagus.
Deficits:
ANATOMY

Lesion of nucleus ambiguus results in atrophy (lower motor neuron) and paralysis of innervated muscles, producing
nasal speech, dysphagia, dysphonia, and deviation of the uvula toward the opposite side (strong side).
No affection of the Sternocleidomastoid or Trapezius. These muscles are innervated by cells in the rostral spinal cord
(caudal portion C.N. XI).

Ans. D: XII
Ref.: BDC 4th ed., vol.3, page-324

166. Ans. C: L3
Ref.: Clinical Anatomy-Snell, 8th ed., page-867

167. Signs of Cerebellar Disorders


i. Ataxia, Reeling, wide-based gait- decomposition of movements, Inability to correctly sequence fine, coordinated acts
ii. Dysarthria-Inability to articulate words correctly, with slurring and inappropriate phrasing
iii. Dysdiadochokinesia-Inability to perform rapid alternating movements
iv. Dysmetria-Inability to control range of movement
v. Hypotonia-Decreased muscle tone
vi. Nystagmus-Involuntary, rapid oscillation of the eyeballs in a horizontal, vertical, or rotary direction, with the fast
component maximal toward the side of the cerebellar lesion
vii. Scanning speech-Slow enunciation with a tendency to hesitate at the beginning of a word or syllable
viii. Tremor-Rhythmic, alternating, oscillatory movement of a limb as it approaches a target (intention tremor) or of
proximal musculature when fixed posture or weight bearing is attempted (postural tremor)
Asthenia: (Weakness/Lack of energy and strength/Loss of strength).
It is a prominent part of myasthenia gravis

Ans. C: Asthenia
Ref.: BDC 4th ed., vol.3, page-340
Anatomy 67

JOINTS

168. Ans. B: Pubofemoral ligaments


Ref.: Clinical Anatomy-Snell, 8th ed., page-589

169. When the knee is flexed, flexion is limited by the anterior surface of the thigh coming into contact with the anterior
abdominal wall.
When the knee is extended, flexion is limited by the tension of the hamstring group of muscles.
Extension, which is the movement of the flexed thigh backward to the anatomic position, is limited by the tension of
the iliofemoral, pubofemoral, and ischiofemoral ligaments.
Abduction is limited by the tension of the pubofemoral ligament, and adduction is limited by contact with the
opposite limb and by the tension in the ligament of the head of the femur.
Lateral rotation is limited by the tension in the iliofemoral and pubofemoral ligaments, and medial rotation is limited
by the ischiofemoral ligament.
The following movements take place:
 Flexion is performed by the iliopsoas, rectus femoris, and sartorius and also by the adductor muscles.
 Extension (a backward movement of the flexed thigh) is performed by the gluteus maximus and the hamstring
muscles.
 Abduction is performed by the gluteus medius and minimus, assisted by the sartorius, tensor fasciae latae and
piriformis.

ANATOMY
Adduction is performed by the adductor longus and brevis and the adductor fibers of the adductor magnus.
These muscles are assisted by the pectineus and the gracilis.
Lateral rotation is performed by the piriformis, obturator internus and externus, superior and inferior gemelli, and
quadratus femoris, assisted by the gluteus maximus.

Ans. D: Sacroiliac ligament


Ref.: Clinical Anatomy-Snell, 8th ed., page-589

170. Popliteus muscle


Origin: Lateral surface of lateral condyle of femur
Insertion: Posterior surface of shaft of tibia above soleal line
Nerve supply: Tibial nerve L4, 5; S1
Action: Flexes leg at knee joint; unlocks knee joint by lateral rotation of femur on tibia and slackens ligaments of joint

Ans. C: Popliteus
Ref.: BDC 4th ed., vol.2, page-115, Clinical Anatomy-Snell, 8th ed., page-617

MISCELLANEOUS

171. There are three types of cartilage:

i. Hyaline cartilage has a high proportion of amorphous matrix. Throughout childhood and adolescence, it plays an
important part in the growth in length of long bones (epiphyseal plates are composed of hyaline cartilage). It has a
great resistance to wear and covers the articular surfaces of nearly all synovial joints.

ii. Fibrocartilage has many collagen fibers embedded in a small amount of matrix and is found in the discs within
joints (e.g., the temporomandibular joint, sternoclavicular joint, and knee joint) and on the articular surfaces of the
Self Assessment & Review of FMGE / MCI Screening Examination
68

clavicle and mandible. Fibrocartilage, if damaged, repairs itself slowly in a manner similar to fibrous tissue
elsewhere.
iii. Elastic cartilage possesses large numbers of elastic fibers embedded in matrix. It is flexible and is found in the
auricle of the ear, the external auditory meatus, the auditory tube, and the epiglottis. Elastic cartilage, if damaged,
repairs itself with fibrous tissue.

Ans. C: Elastic cartilage


Ref.: BDC 4th ed., vol.3, page-255, Clinical Anatomy-Snell, 8th ed., page-36

172. Elastic cartilage is present in the pinna, external auditory meatus, eustachian tubes, epiglottis, vocal process of arytenoids
cartilage

Ans. B: Trachea
Ref.: Snell‖s Anatomy, 7th ed., p-39; BDC‖s Handbook of General Anatomy, 4th ed., p-50

173. Shunts of simpler structure is found in the skin of nose, lips and external ear; thyroid gland, sympathetic ganglia etc.
Specialized AV anastomosis are found in the skin of digital pads and nail beds
Arteries of kidney are end-arteries
Arterio-venous anastomoses
 It is the communication between an artery and a vein.
 It serves the function of phasic activity of the organ
 Shunts of simple structure are found in the:
ANATOMY

Skin of the:
 Nose
 Lips
 External ear
Mucous membrane of:
 Nose
 Alimentary canal
Coccygeal body
Erectile tissue of sexual organs
Tongue
Thyroid gland
Sympathetic ganglia
 Specialized AV anastomoses is seen in the
Skin of digital pads
Nail beds

Ans. D: Kidney
Ref.: BDC‖s Handbook of General Anatomy, 4th ed., p-115

You might also like